You are on page 1of 54

DAY 1

Know Patient Mgmt part of Mosbyits tricky too!


Know how to diagnose between periodontal vs periapical/irreversible vs reversible,
pulpotomy vs pulpectomyas long as you know the key symptoms I think you will do
fine.
MAKE SURE YOU READ CAREFULLY CUZ SOME OF THE QUESTIONS ARE WORDED
FUNNY (to me at least?!)
Pics I had to ID were:
AOT (Adenomatoid Odontogenic Tumor)=the stem had a 19 yr old female & xray was
in the ant maxilla bet LI & K9 (most common presentation) Mosby pg. 119 Epithelial
odontogenic tumor. Zellballen(whirling pseudocyst). Uniolocular RL appearance and
snow flakes. Impacted Canine. Majority maxilla, females, anterior jaws and over
crown of impacted tooth Differntial Diagnosis: Lateral Periodontal Cyst Mosby pg 117
Unilocular/Mutilocular RL in lateral perio membrane. Most found in mandibular PM
Region tooth is vital. Gingival cyst of adult is soft tissure counterpart of this lesion
Epulis Fissuratum=edentulous ridge with excess tissue Mosby pg 322 hyperplastic
tissue reaction caused by ill-fitting or over extended flange in denture. Tx. Adjust
denture border and use tissue conditioner,surgery is indicated if inadequate response
Stafnes defect (it was called Salivary gland inclusion)=typical pan with lesion under
IA canal Mosby pg. 104 RL of mandible due to invagination of the lingual surface of
jaw located in the posterior lingual of mandible below mandibular canal. Impinges of
mandibular nerve. Also called Static Bone Cyst
AI( Amelogensis Imperfecta)=PA of teeth with open contacts & no diff between
enamel and dentin. 3 types.
1.) Hypoplastic inadequate deposition of emamel matrix
2.) Hypomaturation defect in maturation of enamel crystal structure. Abnormal
hardness. Less Radiodense than dentin. Softer than normal
3.) Hypocalcification- normal enamel matrix and no significant calcification. Affects
enamel only without systemic disorder. NOTE: Radiographs are normal may see
taurodontism on occasion and cant diagnose w/ x-rays alone (ORAL PATH NOTES)
Mosby pg. 175 associated with BELL STAGE (Histodifferentiation)
Note:
Lower border of the nose=ant PA of missing 8 & 9 and pointed to it Mosby pg 135
Lateral wall of the orbit=half of a pan of an edentulous maxilla (idk if that was right
but thats what my answer was) Mosby pg 141
Pyogenic granuloma ( pg 112 mosby.AKA Pregnancy Tumor (1st trimester) due to unbalanced
hormones or Eruptive hemiangioma or granulation tissue-type hemiangioma according to wiki , Not true
tumor, local irritant that causes overgrowth of gingiva tissue, Color ranges from pink, red, purple, can be
smooth lobulated, often seen on the anterior nasal septum, occurs on gingiva 75%, maxillary > mand.
Anterior region > posterior. Can also be found on the lips, tongue, and cheek.
The 3Ps: all have same ccl and different histo 1.) Pyogenic Granuloma 2.) Peripheral Giant Cell
Granuloma- females 40-50, serum calcium test performed associated with MNGCs. Sessile or
pedunculated 3.) Peripheral Odontogenic Fibroma bone or cementum calcifications.
Fibrous dysplasia Pg. 120 Mosby- CCL- More common in maxilla. Affects Children, Radiographic
appearance- diffuse opacity (ground glass- frosted beer mug) Oral Path notes- Histology- Chinese
Characters. 2 types monostotic and polystotic. Associated with 2 syndromes 1.) McCune Albrights ( caf

au lait pigmentation, endocrinopathies (percious puberty, pituitary adenoma, hyperthyroidism, polystotic)


2.) Jaffe-Lichtenstein (no endocrinopathies) Pt. can have facial swelling and expansile of jaws
Gingivitis scale is what type of system? (I put ordinal because it had mild=1,
moderate=2, severe=3 in the stem)
What does the W on the rubber damn clamp stand for? Wingless (luckily I saw
this before & found the answer in our oper txtbk..lol)
Pt lip swells up right after impressions, what caused it? Angioedema, anaphylaxis
(Angioedema may be caused by an allergic reaction. During the reaction, histamine and other
chemicals are released into the bloodstream. The body releases histamine when the immune system
detects a foreign substance called an allergen. The main symptom is swelling below the skin surface.
The following may cause it: animal dander (scales of shed skin, exposure to water, sunlight, cold or
heat, foods (berries, shellfish, fish, nuts, eggs, milk, insect bites, medications (penicillin, sulfa drugs,
NSAIDS, blood pressure medicine (ACE inhibitors) . Hereditary angioedema is rare but problem
with the immune system that is passed down through family. It causes swelling in face, airways, and
abdominal cramping resource http://www.ncbi.nlm.nih.gov/pubmedhealth/PMH0001849/ )
-

Older female w/ widening of PDL and resorption of mand?


Scleroderma (it said diffuse systemic sclerosis) Mosby pg 109 An autoimmunie
multiorgan disease of adults especially women. Fibrous of tissues leads to organ dysfunction.
Oral changes include restriction of orifice, uniform widening of periodontal membrane, and
bony resorption of mandibular margin of mandibular ramus (best seen on PAN). CREST
Syndrome- Calcinosis cutis ( calcium salt deposit beneath the skin), Raynauds phenomenon (
temp change from stress tips of fingers turn blue can be tx with nifedipine, amlodipine,
dilitiazem, felodipine, nicardipine) Eshophageal Dysfunction, Sclerodactyly, Telangiectasis.
Diffuse systemic sclerosis is rapidly progressing and affects large area of skin and one or
more internal organs (kidneys, esophagus, heart,lungs)

Whats the least common congenitally missing tooth? (3rd, Man 2nd, LI,
K9)
Mosby pg 193. Most common congenitally missing tooth is 3rd Molars, mand. 2nd Pm,
Max. LI, and then Max. 2nd PM
-

S, z, ch sounds, do what to the teeth?(I put bring them the closest


together)
o Sibilant/linguoalveolar sounds (s,z,sh,ch,j)tip of tongue contacts
anterior palate/or lingual surfaces of teeth. Determine vertical overlap
&length of anterior teeth.
o Mosby pg 233-324 Fricatives or labiodental sounds f, v, ph are made btw max.
incsiorss contacting the wet/dry lip line of mandibular lip. These sounds help to
determine the position f the incisal edges of maxillary anterior teeth
o Linguodental sounds are those using tip of tongue slight between max and mand teeth
to say this, that. Those. If the tip of tongue is not visible the teeth most likely too
anterior EXCEPT in Class II malocclusion or excessive vertical overlap where the
tongue extends too far out and teeth too far lingual
o The B, P, M sounds are made by contact of the lips.

Pt wearing a denture for 19 yrs, wants a new one, you notice a


6x3cm white lesion, what do you do? Biopsy, refer to head & neck cancer,
adjust denture (IDK but thats what I put) Its incisional biopsy bc the lesion is larger

than 1 cm. if it was less than 1 cm you would do excision. Mosby pg 95: Different types are
1.) cytology smear 2.) aspiration 3.) excision 4.) incision
-

Pt has denture for 14 yrs, you notice red raised papules on palate,
what is it? (Inflammatory hyperplasia of palate, nicotinic stomatis may
have red and gray spots))
o IPH-usual cause under ill- fitting denture, denture movement irritation
and/or accumulation food debris. Presents as painless, firm pink and
red nodular proliferations of mucosa. Hard palate usu., may involve
residual ridges. Not completely reversible but can regress w/ smllr
papilla and tx (remv denture, soft relines, good oral hygiene, and
nystatin therapy. Pt. needs to soak denture in 50% water 50% bleach
Pt is on Propanolol for HTN, you give 10mg/ml EPI, what happens? (inc
bronchiodilation, inc HR, inc BPI think I chose inc BP) Propanolol beta
blocker. (b1). Epi is a vasoconstrictor causes slight elevation.

What drug doesnt cause gingival hyperplasia? (Dilantin phenytonionanticonvulsant, cyclosporine( gin graft/ neural)- prevent transplant rejection
nifedipine (Procardia)- calcium channel blockers, Verampil- calcium channel
blocker, a random drug)

Where to place the implant in relation to the adjacent teeth? (asked


me this 2x in 2 diff ways) I put below adjacent CEJ both times (Emergence
Profile- 2-3 mm apical to the adjacent tooth CEJ)

What the most effective brushing technique? (stillman, sulcular, and two other
choices.I chose sulcular bcuz I thought it was the bass technique) (Correct The
Bass Method is also called the Sulcular Vibration technique.. named after Dr. Charles Bass.
This techniques is when the toothbrush bristles are placed at the gingival margin at 45 degree
angle t o the tooth allowing bristles to extend into gingival sulcus when pressure applied. Mosby
pg 268

If you have light ortho force, bone resorption is where in relation to


the force? (undermined, direct, indirect) Mosby pg. 156 Light forces causes smooth
continuous movement without formation of hyalinized zone in surrounding PDL. The teeth
start to move earlier in a more physiological way than with heavy. Heavy force actually delays
tooth movement by causing a lag period after the initial movement of tooth within PDL.
Undermining Resorption occurs within the alveolar bone in the marrow spaces and moves toward
the PDL area. It is associated with heavy force. Appearance of osteoclastic cells in bone marrow
spaces is the first indication of undermining resorption.

What cells are found in the established lesion of gingivitis? (Plasma


cells, macrophages, lymphocytes, and another one) Mosby pg. 248
1. Initial Lesion- 2-4 days, PMNs, Blood vessels is vascular dilation, collagen perivascular
loss, and CCL findings gingival fluid flow
2. Early Lesion 4- 7 days ,Lymphocytes, Vascular proliferation, collage increased loss around
infiltrate and CCL findings erythema and bleeding upon probing and
3. Established Lesion- 14-21 days, Plasma Cells, vascular proliferation and blood stasis,
continued loss in collagen and CCL findings is changes in color, surface, texture Advanced
Stage characterisitics of stage III move into periodontal ligament space and create
periodontitis.

List the orders of Perio therapysaid it like Emergency, plaque control,


reeval, surgical, maintanenceshit I cant remember but you will recognize it
(Mosby pg 252.

1) Preliminary Phase (tx of emergencies, ext of hopeless teeth)


2) NONSURGICAL PHASE (phase 1 therapy- plaque control, diet control, removal of calculus,
excavation of caries, antimicrobial therapy, minor ortho, provisional splinting)
3) SURGICAL PHASE (phase II perio including implant placement and endo therapy)
RESTORATIVE PHASE (phase III final restorations, Fixed, RPD, evaluation of response to
procedures periodontal examination)
4) MAINTENANANCE PHASE (phase IV perio rechecking)
-

In trisomy 21, you will see all the following (LARRY?!...lol)EXCEPT?


Rampant caries (you will see perio disease though) Trisomy 21=Down
Syndrome ( no gingivitis, but periodontitis, very loving patients who
like to hug on you, Provetella Intermedia organisms(orange complex)
are associated with these pts.

NOTE: Prevetella Intermedia organism is associated in pregnancy and necrotizing


perio diseases and down syndrome.
RED complex organisms ( P. gingivalis, T.forsytheia, T. denticola) Dentists for Red
Gingivitis! (so we can get paid!)
Orange Complex (P. Intermedia, F. bacterium, Camphylobaceter) Camping In back
(woods) with Orange fire
-

What symptom will a person with trisomy 21 have? Small midface but
worded diff

What systemic antifungal will you give a person w/ oral candidasis w/


HIV? (Fluconazole was the only system one listed), or Ketoconazole or
Amphotercin B. , Caspofungin. Note: regular oral candidiasis infections are
treated with nystatin rinses or clomatrizole troches. DOC for HIV pts with
candidi is FLUCONAZOLE!

Woman allergic to pen, and needs extractions, what to give before?


(600mg clinda 1 hr beforethe other choices were the right drugs but wrong
doses)
SBE Prophylaxis Mosby pg 308
Look at cephalosporin chart Mosby pg 307
8 month pregnant lady gets extractions, flap, and osseous
contouring?! (something that was gonna cause some post op
pain)what can you give? (acetaminophen 325mg, Tylenol #3, and 2 others
that seemed like NO-NOsidk but I chose Tylenol #3 dammit cuz it seemed
like the right thing to do..lol) Note: Pregnant women can have tylenolol,
propofol, codeine, propoxyphene. CANNOT GIVE warfarin, NSAIDS,
methotrexate, merpidine, nitrous, barbituates. Tylenol #3 contains 300mg
acetaminophen and 30 mg of codeine. Tylenol #4 contains 300 mg
acetaminophen and 60 mg of codeine.

What causes tardive dyskensia? (phenothiazine) Tardive dykensia is


involuntary movement of the muscles. Mosby pg 281 Phenothiazine is an
antipsychotic. Indications schizophrenia other types of psychosis, tourettes
syndrome, hungtingtons chorea, and other disorders like obsessive
compulsive disorders. Adverse motor effects of antipsychotic drugs acute
dystonias, akathisia, parkinosonism, perioral tremor, malignant syndrome and
tardive dyskinesia or extrapyramidal.

What causes extrapyramidal stimulation? (antipsychotic-phenothiazine)

How do sulfonylureas oral hypoglycemics work? Stimulate the release of


insulin from GGGotta gave insulin! pancreas Mosby pg 303 Glyburide,
Glipizide, and Glimepiride. Know Insulin table on pg 303
-

A person on chronic glucocorticoids, you see all the side effects


except? (Hypoglycemic, weight gain, osteoporosis, and 1 more) Pg 303
Mosby Adverse Effects insomnia, agitation, infections, hypertension,
atherosclerosis, skin and mucosal atrophy, negative calcium balance
(osteoporosis) muscle wasting, obesity, glucose intolerance, peptic ulcers, and
cataracts

Which symptom with you find with corticosteroid treatment?


Osteoporosis

The main effect that causes osteoradionecrosis? Something about


effects the vascular supply

Pt on IV bisphosphonates and has carious and non restorable teeth,


what is the best treatment? (I put the answer that said discontinue for 3
months bcuz they didnt have one about talking to the physician) IV
Bisphosphonates ( Zolendroic Acid (Zomate), Pamidronate(Areida)) Oral Drugs
(Fosamax (aldrenote), Actonel(Risedronate) and Boniva (Ibndronate).
Bisphosphonates can cause osteoradionecrosis of the Jaw so treat with
hyperbaric oxygen during dentoalveolar surgery. Note: IV bisphosphonates
have a half life of like 14 YEARS!!! If you need to do anything, you just cut the
crown of at the gingival and do RCT. Do not EXT unless pt is well aware of the
riskseven then, be careful.

Pt on takes a aspirin, how long does it inhibit the platelet activity? (1


hr, 1 week, 24 hrs, 1 monthIDK) Board busters pg 176 discontinuation of
aspirin for 5 to 7 days allows normal clotting time to reapprear due to
synthesis of new platelets. Depending on the frequency and dosage depends
on how long it takes to inhibit the platelet activity

What does Warfarin inhibit? The vitamin K dependent clotting factors


Clotting factors are II, VII,IX, X. Warfarin and Coumadin are vitamin K
antagonist.

Pt had a injection of 4% PRILOCAINE, and had cyanosis of lips and


area of injections, what happened? (I chose METHEMOGLOBINEMIAcuz
that was always the damn answer when it came to prilocaine)

A drug NO INTRINSIC activity, what is it? (agonist, antagonist) Mosby pg


271

Which structure is involved in a Leforte I fracture? Maxillary sinus


Mosby pg 83

What mandibular fracture will cause parasthesia? (Symphisis, coronoid


process, condyle, body) Most common fractures is condyle, angle, symphisis,
body

The advantage over distraction osteogenesis vs split osteotomies


(you could probably pick the right answer after reading the choices) Mosby
pg83-82-83. BC of Bone growth with distraction osteogenesis

What structure is similar to an epiphyseal plate? Synchondroses Mosby


by 146 Sychrondoses become inactive. Intersphenoid age 3, Spheno-ethmoid
age 7, and spehno-occipial considerly.

The most common between cleft lip/palate, ectodermal dysplasia, AI, DI,
and OI? Cleft palate higher in females and is most common in Native
Americans and Cleft Lip is higher in males. Cleft Lip/Cleft Palate is more
common in men and more unilateral and unilateral is more prevalent in
women and bilateral more prevalent in men

? Race more prone to perio disease in US? (Black male, White male,
Asian female, native American female)

Around menarche, you will see what? (sexual maturation, dental


maturation, something else, and more prone to perioI chose it bcuz of the
common time for LAP) Note P. intermedia organism more prevalent in
pregnancy, puberty, necrotizing perio, and down syndrome patients
Prevetella got pregnant during puberty

What stage of histology can you tell the diff in size and shape of
tooth formation? (proliferation, histodifferentiation, morphodifferentiation
got my ass?! Look it up) Mosby page 175-176 answer is Morphodifferentiation (Bell
Stage)- peg laterals, macrodontia. Proliferation (Cap Stage)-supernumerary teeth,
anodontia, cysts, odontomas Histodifferentiation (Bell Stage)- AI, DI, OI Appositionincomplete tissue formation-i.e. enamel hypoplasia and Calcification- localized infection,
trauma, fluorosis, tetracycline staining. So the order of tooth development.
Initiation (Bud Stage) absence of this is anodontia, excessive bud is supernumerary teeth.
Proliferation (Cap)
Histodifferentiation/Morphodifferenitation(Bell Stage)
Apposition
Calcification

1.
2.
3.
4.
5.
-

What is the internal connection (hex) between the implant and


abutment designed for? I chose to prevent rotation of the tooth (sounds
like antirotation to me)

9 yr old pt chipped ant porcelain veneer but wants it fixed not replaced,
how do you prepare? (had to put in order.acid etch, silanate, bond resin,
pumiceI guessed) Brush porcelain 1.etch onto the inside surface of the veneer with the small
brush included with the etch. Brush up and down across the entire veneer to create a rough surface,
which is what will allow the porcelain silane adhesive to adhere better. Allow the etch to dry for 10
minutes.2.Brush porcelain silane onto the inside surface of the veneer with the brush included with
the silane. Brush in a side-to-side motion across the entire veneer.Place the veneer back on your tooth
carefully, easing it slightly under your gum. If the veneer is fractured, squeeze the two sides together
after the veneer is on your tooth to close the fracture.Hold the veneer on the tooth with your fingers
for a minimum of three minutes. Wait four hours before eating to ensure the veneer has bonded
securely to the tooth.
-

Pt wanted veneers and his teeth bleached, what order would you do
it? (I chose bleach, 2 week break, cut teeth cement veneers)

Ant PFM looks too opaque in the incisal 1/3, whats most likely the
problem? (inadequate biplane reduction)

Whats the posterior palatal extent of a complete denture? (hamular


notch, vibrating line, fovea palatineI chose vibrating line bcuz it said
posterior PALATAL extent) Mosby pg 324 The posterior limit extends to
jxns of moveable and immovable tissue. This coincides with the line
drawn through the hamular notches and approx. 2mm posterior to
fovea palatine (vibrating line)

Why would you reduce the opposing dentition before doing a fixed
bridge? Supraeruption

The difference between 245 and 330? 245 longer than 330 (its obvious)

Patient on Warfarin? What test do you order? INR ( International Normalized


Ratio used to monitor the effectiveness of blood thinning drugs such as warfin.
Recommended valuses 2-3 or 2-4. PT (Prothrombin Time- 11-13 secs in normal person but
12- 18 in patients on warfin), PTT (Partial Prothrombin Time 25-35 secs. In normal ppl.
Test of hemophilia, bleeding time less than 9 minutes)

What is the minimum distance that you need between implants? 3mm
(Note: 1mm btw implant and non-implant tooth)

What is the angle of the curette when doing SRP? (20-40, 45-90, 90120)

Where primate space located? (had two columns, max=LI, K9,


man=K9,1stM)

Pt has multiple osteomas, colonal polyps, and dermal cysts, what is


the syndrome? Gardners Mosby pg. 123 autosominal dominant,
supernumery teeth. Intestinal polpys have a very high rate of malignant
conversiton to colorectal carcinoma.
Whats the important factor in DFDB graft that gives it osteogenic
properties?( BMP) (Note DFDB is osteoinductive and FDB is osteoconductive. BMP
stands for Bone Morphogenetic protein)

What is not a factor sodium hypochlorite? Chelating agent(its nonchelating and organic)

Most common location for a siaolith? Submandibular duct

What syndrome will you see multiple OKCs? Gorlin or Nevoid Basil Cell
Syndrome hereditary autosomal dominant multiple basal cell carcinoms of the
skin, multiple OKS, bifid ribs, frontal bossing, calcifications of falx cerebi,
palmer and plantar pitting.

Pt has a new PFM place with overhang, what is the first symptom you
will see? ( I just remember gingival inflammation and gingival recession)

Pt has probing depth of 5mm, the CEJ is 2mm coronal to the free
gingival margin, what is the depth? 7mmI hope?! Lol

How do you measure the periodontial attachment? (I took it as they


were asking the clinical attachment level, a fixed, CEJ, point to the bottom of
the pocket)

How does Zantac(Ranitidine) work? Inhibit gastric acid secretion

Which impression material is not used for doing cast impressions? (I


was in between reversible & irreversible but I thought of DR Tyus in the review
when he said the best impression material is Agar Agar, reversible, so I chose
Irreversible)

Pt had a rough, white, pedunculated nodule on palate, what is it?


(papilloma)

Why is a pulpotomy on a 6 yr olds perm 1st molar the treatment of


choice?(choses were for apexification, physiological growth of root, to help 2nd
molar erupt..i chose my answer bcuz it sound like apexogenesis and you wud
still have vital pulp in roots)

What does primary tooth discolor after trauma?(diffusion of blood in


dentin tubules, fibrosis of pulp, 2 more choices)

What type of occlusion precedes molar class II occlusion? Distal step


class I is flush terminal plane, class II is distal step and distal step is least
desirable and class III is mesial step

Stidor is seen in what? (bronchospasm, laryngospasm, and to other random


choices)(High pitch wheezing that is inspiratory) Tx with succinylcholine High
pitch wheezing expiratory is asthma) Tx asthma with albuterol, theophylline or
epinephrine.
Most common salivary gland tumor? Pleomorphic adenoma Mosby pg 115
Most common malignat salivary malignancy in minor and major glands is
Mucoepidermoid Carcinoma.

What is the treatment of choice for a tooth that is showing inflammatory


external resorption? Pulpectomy w/ Caoh2 mosby 24 via internet External
inflammatory resorption and replacement resorption are complicating factors that
may result from traumatic dental injuries when the tooth is luxated or avulsed and
replanted. Resorption may, ultimately, result in loss of the tooth. However, with

appropriate treatment, the prognosis for these teeth is greatly improved, with the
possibility of preventing or arresting resorption. EXTERNAL ROOT resorption
occurs more frequently than internal root resorption and is
commonly misdiagnosed as internal root resorption. External root
resorption is caused by an injury to the external root surface. After
an injury such as concussion or subluxation, cementum can be
damaged, resulting in a localized inflammatory response and area of
resorption. In about two weeks, the periodontium and root surface
should repair spontaneously, and in that case no treatment is
needed. With severe injuries, such as intrusion or avulsion
(especially when implantation is delayed more than 60 to 90
minutes), active external inflammation can persist and histologically
there will be multinucleated osteoclasts resorbing the dentin of the
root. Seven to ten days after the injury, it is recommended to treat
the tooth endodontically by placing calcium hydroxide in the canals
long-term and replacing the calcium hydroxide in one month and
then at three-month intervals until the resorptive process ends. The
high pH of the calcium hydroxide seems to permeate through the
dentinal tubules thus killing bacteria and neutralizing endotoxin,
which stimulates inflammation. If bacteria are thought to originate in
the sulcus of the tooth (totally external), a vitality test will respond
positively, but in cases where infected pulp causes external root
resorption usually in the apical or lateral aspects, a vitality test can
be negative.
The cause of internal root resorption is unclear, but
trauma and the extreme heat produced when using a high-speed drill
without water have been suggested. Histologically, there is normal
pulp tissue transforming into granulation tissue with giant cells
resorbing the dentinal wall, and resorption will only occur if the
odontoblastic layer and predentin are lost or altered. Internal root
resorption usually contains some vital pulp and gives a positive
vitality test; however, since necrotic pulp tissue is usually found
coronal to the active resorbing cells which are more apical, the tooth
can sometimes test negative. Internal root resorption resolves with
root canal treatment because the resorbing cells will no longer have
the blood supply to survive. In cases where internal root resorption
causes buccal or lingual perforation, mineral trioxide aggregate
(MTA) can be used to repair the site.
You use a complimentary color on a pfm, what does it do? (decrease the
valueblue is the color use to decease the value which is the complimentary
of yellow)
Doing RCT on max 1st molar, what is the surface on the MB root that is most
common for strip perforation? (mesial, distal, buccal, lingual)
The latency of radiation therapy is between? (therapy & film development,
therapy & symptoms, 2 more) Mosby pg 130 so I think therapy and film
development
Which has the greatest cariogenic potential? (sucrose, lactose, glucose,
fructose)
Opiods cause vomiting but? Stimulation of the medullary chemoreceptor
trigger zone
What syndrome has congentially absent teeth? Cleidocranial dysplasia, downs
syndrome, ectodermal dysplasia (it was an obvious choice) Ectodermal
dysplasia ppl have thin sparse hair, anodontia/oligodontia, and dry scaly skin
What is the best way to clean an interproximal space? (interproximal brush,
toothpick, water irrigator)

How much do you take off of the middle 1/3 of the facial surface when
preparing an anterior veneer? (.3, .5, .8, 1)
A needle tract infection following an IANB caused and infection in what space?
(Sublingual, submandibular, temporal, pterygoid)
Trismus is usual caused by infection in what space? Masseteric
What muscle helps mold the lingual flange of the mandibular denure?
Mylohiod, geniohyoid, masseter, ..) PMS-G (Palatoglossus, mylohyoid,
genioglossus, and superior constrictor muscle) all contribute
When removing the internal oblique ride, what is the risk associated?
Pt has a fracture of the right body of mandible, where should you also check
for a fracture? Left condyle Concept whatever side body of mandible you hit
the opposite side condyle is in jeopardy for fracture
What is the best way to look that the TMJ?(CT, MRI, pan,..)
When is it appropriate to do an I&D? (diffused cellulitis, large firm, flucuant
localized,)
What is the most common side effect of N0-O2 sedation? Nausea
The soft palatal is supplied by what structure? (pharyngeal n, nasopalatine a,
greater palatine n & a, lesser palatine n & a)
What should you do after a pt has a larger swelling after initial antibiotic
therapy? Take culture
During extraction, you get a 2mm, sinus exposure, what it the treatment of
choice? (observe, graft, take xray & Rx antibiotics)
Pt is infected with MRSa, what antibiotic do you give? Vancomycin mosby pg
305 & 307 MOA- inhibits transglycosylase in cell wall synthesis. Narrown
Spectrum. Gram positive aerobe and aneroble. Given IV. Indications for
staph.aureus including methicillin resistant staphylococci, Strept, Enterococci
and Clostridium Difficule. ADVERSE EFFECTS- rental toxicity, ototoxicity, and
RED MAN SYNDROME
Infection in the upper lip can go where? Cavernous sinus
Why is upper lip infection serious to treat? Cause facial veins lack valves
What is the earliest sign of cavernous sinous thrombosis? (Periorbital edema,
decreased vision, ophthalmoplegia) Cavernous sinus thrombosis symptoms
include; decrease or loss of vision, chemosis, exophthalmos (bulging eyes),
headaches, and paralysis of the cranial nerves (III, IV, V, VI) which course through
the cavernous sinus. This infection is life-threatening and requires immediate
treatment, which usually includes antibiotics and sometimes surgical drainage.
What do you seen first with healing after SRP? Long junctional epithelium
Which drug is an antifolate analogue? Methotrexate (used for cancer tx)
Reduction of cusp is determined by what factor? (outline, convenience,
resistance, retention)
-dose ratio of a drug is dependent on what factor? (clearance, distribution
idk)

KRISTIN:
Hope this help Ill post more as I remember Good luck
1. Posture of patient affects what? VDO VDR protrusive, CR, etc
2.

HSV 1 what % US population? IDK I put 30% look it up Per Wikipedia, In the
US, 57.7% of the population is infected with HSV-1[68] and 16.2% are

infected with HSV-2. Among those HSV-2 seropositive, only 18.9% were
aware that they were infected.[69] Worldwide rates of HSV infection
are between 65% and 90 determined by the presence of antibodies
against either viral species.[67]
3. Patient feels helpless in chair how can you help? Let them raise hand when
needed
4. What is seen in diabetics? Murcomycosis Aka Zygomycosis. Found in insulin
dependet diabetics w/ ketoacidosis, transplant recipients, and
chemotherapeutic patients. Murcomycosis is opportunistic fungal
infection and found in bread molds, and decaying fruits or vegetables.
Symptoms nasal obstruction, facial swelling and visual disturbances.
If untreated leaves black and necrotic tissue in palate in maxilla.
Histo: nonseptate hyphae branching at right angles. Tx is surgical
debridement and systemic administration of Amphotericin B
5. Tooth erupts through tissue? roots complete
6. Patient had flu shot never had a dental shot but scared as shit cause of flu shot
what is this? Generalization I think KAPLAN HANDOUT_ GENERALIZATION- a
different but similar stimulus brings on a result. i.E. White uniform,
handpiece/prophy. EXTINCTION- loss of a response after several times
with no reinforcement. Extinction of a behavior or dental fear
7. Patient kept having failure with veneer 3 times why is this? I put something with
anterior guidance off why he kept fracturing them IDK Dental Board Busters
pg 327. Contraindications of Porcelain Veneer- severe imbrication of
teeth, traumatic occlusal contacts, unfavorable morphology,
insufficient tooth structure and enamel. A patient with high caries
index, short ccl crown, and minimal horizontal overalp.
8. Porcelain veneer greater than composite veneer except I put 2 appointments
more time to do
9. Implant and something about fixing with angulations? I chose something with
use angled abutment cant remember
10.How do you know if its perio over endo? Pain on lateral percussion
11.Maxillary permanent incisor had hypoplasia when did it occur? Utero, 1 month
to 6 monthetc know calcification Mosby pg 176
Time
Birth
6
months
12
months
18
months
24

****CALCIFICATION TIMES PERMANENT TEETH


Teeth That Begin to
Calcify Permanent
First molars
Anterior teeth EXCEPT
Max. LI
Max.LI
First PM
Second PM

TOOTH
1M
CI
LI
months
30
months

2nd Molars

C
1PM
2PM
2M

MAXILLA
BIRTH
3-4
MONTHS
10-12
MONTHS
4-5
MONTHS
1.5 YEARS
2 YEARS
2.5 YEARS

MANDIBLE
BIRTH
3-4
MONTHS
3-4
MONTHS
4-5
MONTHS
1.75 YEARS
2.25 YEARS
2.75 YEARS

CALCIFICATION OF PRIMARY TEETH


TOOTH
CALCIFICATION
CI (A)
14 WEEKS IN
UTERO
1M (D)
15 WEEKS IN
UTERO
LI (B)
16 WEEKS IN
UTERO
CANINE (C)
17 WEEKS IN
UTERO
2M (E)
18 WEEKS IN
UTERO

TOOTH
ERUPTION
CI
LI
CANINE
1PM
2PM

MAXILLA

MANDIBULAR

10
11
19
16
29

8 MO.
13 MO.
20 MO.
16 MO.
27 MO.

MO.
MO.
MO.
MO.
MO.

NOTE: CALCIFICATION : ADBCE AND ALL PRIMARY TEEETH BEGIN TO CALCIFY IN


UTERO AND ERUPTION: ABDCE WITH TEETH BCD ERUPTING EARLIER IN MAXILLA
12.Post insertion denture case man had generalize pain on ridge? Not enough
interoccusal space the VDO is too great
13.Anxiety patient what drug can you give the night before to help sleep?
Diazepam, clonazpam, ambienetc AMBIEN (ZOLPIDEM and ZALEPHON) is
sedative hypnotic drug that works similar to benozodiazepines but on
the BZ1 receptor type. It is a long-acting drug and is taken orally. It is
used to treat insomnia and difficulty falling asleep. Benzo drugs
enhance the effect of GABA at receptors on CL channels.
14.Angles Class 1 occlusion what line up with what? Mesiobuccal cusp of 1st max in
the buccal groove of 1st mandneed more question like this
15.Child afraid of dentist and has never been who fault is it? I put parent a
subjective condition
16.Patient asks about Mask gloves and eyewear whats the least response to say?
Something reckless but you will know once you see it.
17.Most dental treatment for the Aged is funded by who? I put out of pocket
considering 60-70% of U.S. dont have dental insuranceMedicare does not
cover dental only Medicaid Mosby pg 221 Medicare- insurance for elderly
and disabled Americans. DOES NOT cover dental care except when
dental services are directly related to the treatment of medical
condition. i.e. extraction of teeth prior to radiation therapy for cancer.
MEDICAD- mandates that states provide early periodic screening
diagnostic treatment for children. It has optional adult coverage, but
varies by state for benefits. CHIP- is called Children Healths insurance
program and it provides basic preventive, diagnostic, and restorative
services but not as broad as medicad
18.Radiation to the operator is mostly from? I chose scatter for patient body IDK
choices where stupid
19.With removal of internal oblique ridge what could happen? I chose possible
damage to lingual nerve
20.The Dr. lavell stuff far as patient management is common sense but the cohort
retrospective cross sectionaletc is very tricky and worded very crazy just try
to know it cold I thought I did but clearly.. MOSBY PG 310-311
21.Pregnant woman with hypertension? Preeclampsia
22.ASA II
23.Head and neck radiation what caries do you see most? I chose cervical VIA
INTERNET Radiation or cervical caries are usually related to xerostomia
and/or chemical changes in saliva. Radiation therapy to the head and
neck region usually exceeding 4000 rads.is the most common cause

24.Lady had white cloudy whatever on buccal mucosa disappear when stretched?
Lukoedema White lesion that is bilateral opacification in the buccal
mucosa. It has a whitish grey appearance that returns to normal when
skin is stretched. It is seen in dark pigmented skin people and those
who smoke
25.Patient listens to favorite music on headphones during treatment what is this?
Distraction, desensitizationetc Mosby pg 228. Treatment of managing
anxiety. Distraction-giving the patient a competitive attentional
focus(listen to music, watch tv), desensitization- exposing a patient to
items from a collaborative hierarchy of slowly increasing anxiety
provoking stimuli)related to target fear) while using relaxation skills.
Others concepts to relieve anxiety are 1.)diaphragmatic breathing 2.)
muscle relaxation 3.) Guided imagery- patient uses diaphragmatic
breathing skills while imagining a pleasant scene of their choice 4.)
hypnosis 5.) behavioral rehearsals- pt. has opportunity to practice
coping strategy using diaphragmatic breathing while experiencing a
simulated procedure or part of a procedure.6.) flooding- intense or
prolonged exposure to a feared stimulus while using coping skills 7.)
biofeedback- teaching one to have control over his/her physiological
arousal through the use of auditory/visual monitoring. 8.) cognitive
coping (reframing) assisting pts. With changing their thinking 9.) use
praise- asking patients to practice coping skills at home and when in
office.
26.Non-working questions like 8 of them just saying thingLingual inclines of
buccal on mandible and inclines on lingual cusp of maxillary
27.Modified ridge lap pontic should be passive very little contact to ridge Mosby
pg 332 -333. A pontic design is in 2 categories 1.) Mucosal ponticsridge lap, modified ridge lap, ovate, conical or bullet shape. All of
these should be concave and passively contact rige. 2.) Nonmucosal
pontics sanitary (hygienic) and modified sanitary hygienic. These are
generally in nonesthetic areas. A saddle pontic design covers the
ridge labiolingually forming a concaeve area that is uncleansed and
for that reason is not used.
28.Bleeding time measure what? Platelet clotting, intrinsic factors, extrinsic factors,
common pathway
Bleeding time=The time it takes for bleeding to stop (as thus the time
it takes for a platelet plug to form) is measure
29.NSAID affect what?
NSAIDS affect the prostaglandin production,
30.Aspirin and asthma is a no go!!! Cant remember the? Thats the ans
31.Free gingival graft what can you eff up? I chose greater palatine neural
bundle.most of the grafts come from hard palate
FGG is the removal a section of attached gingival from another area of the
mouth usu. hard palate or edentulous area to the recipient site. FGG is used to
increase the zone of attached gingival and gaining root coverage.
Success is when graft in immobile at recipient site.
FGG is dependent the blood supply of it recipient site.
FGG failure mostly due to disruption of the vascular blood supply before
engraftment. Infection is second most common reason of failure.
FGG indications:
prevent further recession and successfully increase width of attached
gingival.
Cover non pathologic dehiscences and fenestrations

Performed with frenectomy


Cover a root surface
To widen attached gingival after recession
32.Acute asthma patient his bet blocker is not working what else can give? Epi I
think
33.Non odontogenic pain how can you tell? Give a block if patient is still in pain its
not the tooth
34.What injection will most likely cause hematoma? PSA
35.The elderly suffer from depression
36.ANB is a -6 what is it? I chose Class3 most of the time if ANB is negative its class
3
37.After insurance pay their part the remaining that the patient pays is the? IDK
deductible seem like the only one that made sense
38.Salivation with denture effects retention more is better
39.What is main purpose of splinting teeth? Patient comfort
40.Someone had a delayed hypersensitivity reaction what can you give? I chose
Diphenhydraminethere where two questions like this other ans was
antihistamine same thing
41.Whats a contraindication in doing a RCT? vertical root fracture
42.Main factors when doing a posterior composite? I think I chose type of resin and
depth of prep idk cant remember
43.What is seen in DI? Kind of tricky had two possible ans but I went with
obliterated pulp think the other choice had something to do with osteogenes
and blue sclera
44.What will happen if you get to happy and add to much monomer with acrylic?
increase shrinkage
45.Reason for mucocele on lip? Obstruction, minor salivary gland by mucus plug
46.Which patient is most likely to have paint in joint? Sickle cell, hypertension...Etc
RA was not a choice
47.Which condition is most likely to have TMJ anklyosis as well? Rheumatoid
arthritis
48.Purpose of Plaque Index? Patient motivation seem like the best choice out of the
bunch idk
49.Why do you use irrigation when placing implant? Keep from overheating
50.5 yr old child is acting out how do you stop it? Whip dat ass, naw I think Voice
control for the boards
51.Autistic child-likes affection, needy, repeat things over and over
52.Patient had very old large Mod amalgam with margin intact but pain when
trying to bite a piece of French bread and cold sensitivity?
53.Patient had oral Candida all are possible questions to ask except? I chose
something about him eating bread with no preservatives.
54.30 plus yr old patient had deep fissures and pits what do you do? Observe
55.Had a question of yield strength and ultimate strength dont remember but
know them
56.Where can you most likely see a nutrient canal on an x-ray? Ans choices where
all teeth except the molars IDK
57.All the following are true of a schedule 2 drug except?
58.Best way to treat retained roots in over denture? I chose something with
microbial (keep them clean)
59.Polyether material is very stiff
60.Dentist biggest concern with a denture patient or something like this? I chose
making sure to get the realistic factors across to the patient about the denture.

61.Trauma to child mandible what is most common? Mandible asymmetry,


hyperplasiaetc.
62.Two question about mouth breathers and whatever with mandible plane angle
63.Question about transferring a wheelchair person for wheelchair to the chair?
Sliding board, ask patient, dont use patient belt, etc idk I chose the last one
WTF you need a belt for O_o
64.Dens in dente mainly seen in which tooth? max lateral
65.Most likely IAN parathesis at which fracture in mandible? I chose angle/body?
66.Dentist is doing MOD on max molar when is it ok to cross the oblique ridge?
When its less than 1.5mm left of tooth structure
67.No calculation for me so glad cause all I know how to count is that money
Pheeeewww
68.Review Perio and know what type of organism are seen with each type including
your endo ones
69.09-10 remember is a good review not going to be the same but its a few I
would def look at it
70.Know INR
71.Had a lot of RW post so this just and add on to that
72.If you dont know the ans go with C..shrug seem to be the ans choice of the
day
73.Too much vertical angulation in xray....distortion, overlap,magnification, and
sharpness ( the put distortion)
74.What is reverse architect in peridontitis?
75.Showed a pic of some 15 yr old boy mouth had excessive gingival inflammation
all of a sudden for 5mos and not on meds also bruise easily? I said he had acute
leukemia but idk
76.Childs heartbeat look at chart in mosby pg 183 I put 110 but the child was 4yo
and that was the best choice everything else was below 100
77.Somebody is freakin out they feel doomed!!!!!! Forgot other answer chioce but I
put panic attack.
Day 1:
Good Luck Hope this helps, this is just from the first day.
Pephigoid, know calcifacations times, what effects xerostomia causes, Papillon le fever, definition of
Papilloma,
Regen of periosteum needs- sharpeys fibers, cementum, alveolar bone, periodontal ligament
Denture pt with opposing teeth? Mx bone resorption, post tuberosity droops, Mx anterior resorption
(Combination syndrome)
What cause angular chelitis-VDO loss
159. Palmar plantar keratosis- Lefevre Papillone
#8 lighter than the rest of teeth what do you do? Bleach other teeth, crown
Pregnant woman in dental chair- lay on left side to prevent from laying on vena
Pt on antidepressant what is your greatest concern? Epinephrine or time in chair
These patients are usually taking MAOI which may potentiate the effect of the Epi by inhibiting the reuptake.
Gardner syndrome- osteomas, polyps that turn into adenocarcinomas, supernumary teeth
H2 histamine receptor is for gastric acid
INR determines PT measure warfarin dose, liver damage, vit k status
T test vs chi square (The t-test assesses whether the means of two groups are statistically different from
each other)

Most: 3rd molarmand 2 bicuspidmax lateral (least)


Fentanyl is an opioid reversed by naloxone and flumazenil reverses benzodiazepine
Purpose of plaque index? Show the patient
Fluoride- how much do we use in community water 0.7-1.2 ppm
Main reason for redoing anterior composite-discolored
Reason for mucocele on lip? Obstruction, minor salivary gland by mucus plug, trauma
The mucocele constitutes the most common nodular swelling of the lower lip. These swellings are
asymptomatic, soft, fluctuant, bluish-gray, and usually less than 1 cm in diameter. Enlargement
coincident with meals may be an occasional finding. The most common location is the lower lip
midway between the midline and commissure, but other locations include the buccal mucosa, palate,
floor of the mouth, and ventral tongue. Children and young adults are most frequently affected.
Trauma is the etiologic agent.
156.
Why is 3 degree burn vs 1 degree burn less painful
First-degree (superficial) burns
First-degree burns affect only the epidermis, or outer layer of skin. The burn site is red, painful, dry,
and with no blisters. Mild sunburn is an example. Long-term tissue damage is rare and usually
consists of an increase or decrease in the skin color.
Second-degree (partial thickness) burns
Second-degree burns involve the epidermis and part of the dermis layer of skin. The burn site
appears red, blistered, and may be swollen and painful.
Third-degree (full thickness) burns
Third-degree burns destroy the epidermis and dermis. Third-degree burns may also damage the
underlying bones, muscles, and tendons. The burn site appears white or charred. There is no
sensation in the area since the nerve endings are destroyed.
Erosion- bullemia
Patients with bulimia may present with severe erosion of the lingual and occlusal surfaces of the
teeth Severe erosion can cause increased tooth sensitivity to touch and to cold temperature. Dental
caries may be more prevalent in these patients. The amount of saliva produced may be decreased.
Patients often report dry mouth. Those with poor oral hygiene have increased periodontal disease.
The parotid gland may become enlarged, and patients with anorexia nervosa may have decreased
salivary flow, dry mouth, atrophic mucosa, and an enlarged parotid gland.
Aspirin: associated with Asthma
Contraindication for implant-myocardial infarct, smoking, bone loss
Periostat n doxycycline inhibits what? collagenase
Ectodermal dysplasia
Hereditary ectodermal dysplasia
1. An X-linked recessive condition that results in partial or complete anodontia.
2. Patients also have hypoplasia of other ectodermal structures, including hair, sweat glands, and
nails.
69. How long do you take patient off of Coumadin before surgery? 2-3 days
The INR is used to gauge the anticoagulant action of warfarin. Most physicians will allow the INR to
drop to about 2.0 during the perioperative period, which usually allows sufficient coagulation for

safe surgery. Patients should stop taking warfarin 2 or 3 days before the planned surgery. On
the morning of surgery, the INR value should be checked; if it is between 2 and 3 INR, routine oral
surgery can be performed. If the PT is still greater than 3 INR, surgery should be delayed until the
PT approaches 3 INR. Surgical wounds should be dressed with thrombogenic substances, and the
patient should be given instruction in promoting clot retention. Warfarin therapy can be resumed the
day of surgery
(Hupp, James R.. Contemporary Oral and Maxillofacial Surgery, 5th Edition. Mosby, 032008. 1.3.6.2).
Warfarin and Coumadin are oral anticoagulants that inhibit the biosynthesis of the vitamin K
dependent coagulation proteins (factors VII, IX, and X and prothrombin). These drugs are bound to
albumin, metabolized by hydroxylation by the liver, and excreted in the urine. The PT is used to
monitor warfarin therapy because it measures three of the vitamin Kdependent coagulation proteins:
factors VII and X, and prothrombin. The PT is particularly sensitive to factor VII deficiency.
Therapeutic anticoagulation with warfarin takes 4 to 5 days. 1
Level of anticoagulation and need for altering dosage to avoid excessive bleeding
PTR (1.5 to 2.0) or INR (2.0 to 3.0): Dosage does not need to be altered
PTR (2.0 to 2.5) or INR (2.5 to 3.5): Dosage may be altered
PTR (2.5 or >) or INR (3.5 or >): Delay invasive procedure until dosage decreased
Decision is made to alter dosage of anticoagulation medication
Physician will reduce patient's dosage
Affect of reduced dosage takes 3 to 5 days
Dental appointment needs to be scheduled within 2 days once desired reduction in PTR or INR
has been confirmed
67. Null hypothesis
the null hypothesis, which is the hypothesis that there is no real (true) difference between means or
proportions of the groups being compared or that there is no real association between two
continuous variables
65. Denture for 19years- relieve pain denture and have white spot what do you do
Relieve the denture in the area of the lesion and reevaluate in 1 week.
47. Incidence 100/1000
Incidence: indicates the number of new cases that will occur within a population over a period of
time (e.g., the incidence of people dying of oral cancer is 10% per year in men aged 55 to 59 in our
community).
37. Nevoid BC
Nevoid basal cell carcinoma syndrome (Gorlin syndrome) is an autosomal dominant inherited
condition that exhibits high penetrance and variable expressivity. The syndrome is caused by
mutations in patched (PTCH), a tumor suppressor gene that has been mapped to chromosome 9q22.3q31. Approximately 35% to 50% of affected patients represent new mutations. The chief components
are multiple basal cell carcinomas of the skin, odontogenic keratocysts, intracranial calcification,
and rib and vertebral anomalies. Many other anomalies have been reported in these patients and
probably also represent
Periapical cemento-osseous dysplasia

1. A reactive process of unknown cause that requires no treatment.


2. Clinical features
a. Commonly seen at the apices of one or more mandibular anterior teeth.
b. No symptoms; teeth vital.
c. Most frequently seen in middle-aged women.
d. Starts as circumscribed lucency, which gradually becomes opaque.
An exuberant form that may involve the entire jaw is known as florid osseous dysplasia
Manic depressive not taking medicine what will happen? Mood swings
17. What turns porcelain green? Copper or silver
Porcelain that is baked onto a high fusing gold alloy may exhibit a green discoloration due most
likely to contamination of the metal by COPPER traces. (Dental Decks)
Examples of metallic oxides and their respective color contributions to porcelain include iron or nickel
oxide (brown), copper oxide (green), titanium oxide (yellowish brown), manganese oxide (lavender), and
cobalt oxide (blue). Opacity may be achieved by the addition of cerium oxide, zirconium oxide, titanium
oxide, or tin oxide
13. LED curing vs regular curing? Why is LED curing beter? Range, last longer
Most recently developed are the LED curing units. These units have a number of advantages
compared to other curing units, including a wavelength spectrum emission that is closely matched
to camphorquinone. In addition, these units are more energy efficient, allowing them to be battery
operated. The diodes have a life span that is approximately 1,000 times longer than the typical
halogen bulb. While the earlier versions of LED curing units provided inadequate irradiance, the
newer generation has overcome this deficiency. About the only disadvantage to these units is their
narrow wavelength spectrum, limiting their usefulness in curing any materials that do not use
camphorquinone as the photoinitiator.
The practical consequence is that curing depth is limited to 2 to 3 mm unless excessively long
exposure times are used, regardless of lamp intensity.
Implants: 3mm from another implant
5: mm from mandibuar canal
? from adjacent tooth
SW:
1.
Which test is MOST valuable in an tooth that needs testing that has an open apexyoung tooth =
cold test (EPT does NOT work)
2.
Internal bleaching will MOST likely cause? Extracanal cervical resorption, replacement resorption,
external resorption (idk? Was not aware of what cervical resorption was especially when given extracanal
cervical resorption AND external resorption in answer choices???)
3.
Apexification-done when tooth is NOT vital (aka need RCT) and you need to close the apex so you
can get a seal for the gutta percha; Apexiogenesis-done when tooth IS VITAL and RCT is NOT necessary

no matter what has happened (but you have to know the situations in which a tooth would be vital or
nonvital)
4.

When tooth is traumatically intruded LET IT ERUPT!

5.
Difference between reversible and irreversible and necrotic SYMPTOMS how long pain lingers to
COLD test etc.
6.

Sensitivity to percussion and biting you know you have acute apical periodontitis

7.

MOST likely cause of pulpal inflammation= DECAY/BACTERIA/CARIES etc.

8.
Pain from which one, mandibular premolar or mandibular molar, refers to the ear? Idk? I have a hard
time choosing between the two
9.
SLOB rule question, Buccal root shot from M, now shoot from D and its oppositie the lingual root
shot from mesial blah blah blah (SAME LINGUAL OPPOSITE BUCCAL)
10. Vertical root fracture= EXTRACT
11. Disadvantage of NaOCl: toxic to tissues (does NOT remove smear layer btw)
12. Difference b/w self etch and total etch: self etch does NOT remove smear layer
13. Bacteria responsible for pulpal infection: not specifically which one but answer choice was one or
more than one bacteria? Idk
14. Ledge bypass the ledge
15. Tooth most likely to have 2 canals: max 1st pm
16. Where canals are in mand molar most likely: 3= 2 M and 1D
17. Which access preparation canals look like a C shape? I forget mand molar?
18. What facilitates RCT NOT calcified canals
19. pH 5.5 critical
20. treatment sequencing Mosby p. 38
21. problem with amalgam lies in environmental disposal
22. difference b/w resistance and retention forms
23. knowing that liners and bases and recurrent decay can all appear radiolucent
24. on a pan 2 bones that can appear below mandible= hyoids
25. place calcium hydroxide then resin glass ionomer base then restoration

26. admix= spherical and boxy particles


27. breakdown of a composite margin most likely due to: shrinkage during curing, expansion, or
overfinished margins? Idk
28. large posterior decay: large composite is NOT ideal
29. COTE = gold< amal< unfilled composite
30. Alveolar osteitis= dry socket
31. Bleeding after ext 3 hours later remove clot to locate location of problem
32. Pt has 3rd molars what do you do: tell the patient 3rd molars cause crowding and need to come out,
do not inform patient about them, 3rd molars are associated with cysts and various pathologies and need to
be extracted, or do not do anything about them
33. Pt has cirrhosis of liver: what is his liver mostly composed of? Hepatocytes, fibroblasts, hematopoetic
cells idk
34. Pan had a 3rd molar that was basically straight up and down maybe tilted to distal by 2 mm but erupted
fully it was NOT distoangular impaction (know what these look like on an xray-distoangular,
mesioangular impaction)
35. Day after ext pt comes back with fever and sick feeling give him different antibiotic (bc he has AA
bacterial infection which is associated with ext is what I thought) OR refer him to proper specialist idk
36. Adult pt has crossbite needs fixin: listed 3 ortho appliances-even quad helix (tricky), or surgery- adult
so I chose surgery
37. Distraction osteogeneis good for LARGE movements
38. ANB 6 Class 2 dental or Class 2 skeletal malformation?
39. Trigeminal neuralgia is NOT associated with a spontaneous dull ache
40. Pt reports 3rd molars have been ext years ago pan reveals small round RO in area of #17: osteitis
41. Pt has cirrhosis of liver: what is best: lidocaine, mepivicaine, bupivicaine, articaine
42. Infected tissue LA will be in ionized form
43. Prilocaine- methemoglobinemia
44. Swelling in front of SCM:
45. Cleft lip/palate 1:700
46. First dental formation: (weird!) 6 weeks, 16 weeks, 32 weeks idk (teeth, palate, what idk what youre
talking about)

47. Syphilis lesion that looks most like herpes? Idk


48. Pemphigoid against basement membrane (pemphigus against something else)
49. PCOD ant aa female ANT MANDIBLE
50. Xray teeth with no pulps: DI pg 124 mosby
51. Target made of tungsten
52. Mosby 141 post wall of zygomatic process on xray
53. Man w/ ill fitting partialsindurated ulcerated lesion lat border of tongue: SCC or trauma from
partial? SCC and biopsy
54. Untreated decay mostly in AA oral cancer mostly in AA males mosby 207
55. Pg 163, 164, 165 know like back of your hand quad helix corrects crossbites, use these all in
GROWING ppl, surgery for adultwhen to use LLA or band loop PLEASE KNOW THIS! LLA-bilateral
loss, #19 and 30 NOT erupted yet= need distal shoe if they are erupted need BL
56. Chi square= categories, ttest=averages or means
57. Modeling pg 225 mosbys
58. Probing + recession= clinical attachment level
59. Do NOT attempt perio surgery until you have tried and failed at initial SRP therapy
60. 45-90 angle on SRP instruments
61. Most cost effective: stress oral hygiene home care
62. Pg 272 intrinsic activity and maximal effect and efficacy and receptors and affinity just know it all and
how its all interrelated
63. Beta blockers end in olol, anti GERD drugs prazoles (omniprazole=tagament/prevacid etc)
64. Pharm: carbamazipene, atropine, mechanism of tricyclic antidepressants
65. If you change vertical dimension occlusion during fabricating a complete denture what do you have to
REDO? CR or facebow CR!
66. How to help pt who gags with their denture? Tell em to put the denture in for as long as they can, put a
spoon and hold it for as long as they can tolerate it-do this over and over-YES
67. Implants 3mm apart
68. 323-324 mosby denture phonetics just know why you make all the sounds ALL of them, esp f,v, ph
(do it yourself and youll remember)

69. Kennedy classifications must know cold


70. Value is lightness or darkness
71. Primary tooth with most buccal and lingual convergenceidk
72. Hemoglobin type in sickle cell disease: A, C, F, or S? idk S
73. Pan with laterals missing in photograph but present on pan just impacted, 9 years old, canines
impacted (#1 dental age conincides with his chronological age-NO #2 dental anomaly in this kid occurs
more in permanent than primary dentition, this occurs in proliferation stage-TRUE OR FALSE idk)
74. Pan with 2, 14, 15 w/ 50% bone loss and bombed out 14 and 2 and unilateral edentulous area on
mand tx plan? A.crown lengthening and crowns B.orthodontic tx on maxillary C.max class 2 partial D.
MAND CLASS 2 PARTIAL
75. Radiolucency on pm with MO amalgam could be all EXCEPT: recurrent decay, liner, base, something
off the wall
76. your fees are high! your response should be? it seems you are concerned if getting your dental care
is worth it? OR my fees are the same as all the other dentists or I have to pay bills blah blah
77. Modeling use sister or other kid to show uncooperative kid what to do
78. Reason for everything: PLAQUE

JAK:
AOT, Ameloblastoma, LPC etcknow coldie location
Implant measurements
Soldering
Flaps
GTR
Systemic fungal antibiotic prophylaxis
Calculate lidocaine for a child
Child head shape
Fontanelles
Pierre Robin
Non-chelatingnaocl
Crevicular fluid stuffnomaldiabetic etc
Insurance stuff
Ethics
Ceramics
Ulcer/herpesknow cold its tricky
Xerostomiacauses
Turodontism/pulp stones?occlusally, apically, both
Most commonsa lot ie impacted
Abnormal eruption
Childhood caries
Cleft palate
Xray/max per year percentage between d
Kvp and ma
Difference between round and rectangular cone beams
Bisphosphinates
Distance (doubled)
Max xray per year/week
Know how to read ceph
AP: Greetings Fellow Docs.. First and foremost, thank you to everyone who posted
there remembered questions thus far.. It helped me
immensely!!!!!!!!!!!!!!!!!!!!!!!!!!!!!!!! I want to let you know that the exam is very fair
and if you have studied you will do well .. I would definitely review the classes
remember questions as well as the 09-10!!!! If you know all of them, you will have a
nice bunch of gimmies before it starts to get real.. LOL, There are some tricky
questions though but put your thinking caps on.
Pics I had to ID were: All From RW test I had his entire exam.. and KGs so I will
only add things that I can fill in the blanks to or that were not present.
THANKS R2!!!! ;-) and KG!!!!!!!!!:-)
-Regional odontodysplasia
A dental abnormality of unknown cause ; genetics trauma, nutrition and
infection have been suggested.Quadrant of teeth exhibit short roots, open
apices and enlarged pulp chambers.
The radiographic appearance of these teeth has suggested the term ghost
teeth.
-Zygomatic process of the Maxilla= pg 141 mosby

PAN point to a radiopacity anterior the poster border of the max


sinus
-What symptom will a person have with trisomy 21 have? Small midface
( deficient midface)
-Why would you reduce the opposing dentition before doing a fixed bridge
Establish adequate occlusion
Remove interferenece before to eliminate reducing them later
. Cant really remember the others at the moment.
-What does primary tooth discolor after trauma?(diffusion of blood in dentin
tubules, fibrosis of pulp, internal resorption, 1more choice)
-When removing the internal oblique ridge, what is the risk associated?
Damage to the lingual nerve, other choices Thats the answer I went. I agreed
with Krisitin!!
-Bleeding time measure what? Platelet clotting, intrinsic factors, extrinsic factors,
common pathway
-after extraction patient is experiencing hemorrhage 3days later. Why?
- vagal something. Sorry thats all I can remember. That is the only answer
that seemed to be the most correct to me.
-Patient has a3mm palate constriction resulting in a unilateral posterior crossbite.
What does he to to maximize is occlusion?
- Functional shift to the side of the crossbite
- Functional shift to the opposite. ( I chose this answer)
- Anterior open bite
- One more selection
-What is the plaster record used for the articulator after mounting the maxillary
cast???
Centric relation
Establish the vertical dimision of occlusion
To not mess up the transfer of the facebow record.
What causes green and orange staining on the anterior teeth.
Diet
Ingestion of metallic substances..
3 other choices.. I chose
What can you give a medically compromised child to help with plaque accumulation.
Fluoride
Chlorohexidine
What does not involve measuring variation:
Range. , variance, standard error , mode
(dont really know what this is,,,, so thats way I selected this. )
Had a lot of questions for patient management. Most I have not seen on other
remembered Qs.
-A test is supposed to test positive for 95 out of 100 cases that contain a particular
disease. This test is considered.
Sensitive, specific, reliable, valid..
----Desensitization questions (5-7 questions)

----Distraction- Patient is listening to headphones while performing a dental


procedure
----OSHA---- like 5-7 questions- what falls under OSHA policies and procedures.
----Components of a scientific article: title, abstract, Introduction, methods results and
discussion.
Had a lot of Removable questions.. What the F**K!!!!! ( thanks daphne!!!!!
NOT)
These questions were worded extremely funny so I really cant remember how
to articulate them..
They asked about distal extension/ distal extension vs. tooth borne RPDs- what is the
different between the distal extension vs. the tooth born RPD.
Bilateral distal extension .. Just know the effects of indirect retainers, rests and
there actions.
KNOW Composites!!!! Questions were funny interpreting the question is 90% of
the answer..
Had a lot o perio related to scaling and root planing,periodontal maintenance, endo
perio lesions, perio-endo lesions etc
I PLAN TO ADD MORE AS I REMEMBER. FOR THOSE TAKING THERE EXAM TOMORROW
I WISH YOU THE BEST.. SORRY I COULD NOT REMEMBER ALL THE ANSWER CHOICES
AND EVEN SOME OF THE ANSWERS I SELECTED. LORD KNOWS I TRIED, BUT
WHATEVER I RECOLLECT I WILL BE SURE TO ADD ASAP!!!
PEACE CREST KIDS!!! AS MY DAD CALLS US ;-)
PC:
Some repeats from class and some from 9-10not a lot. TRICKY QUESTIONS.
Know wht mercury toxicity does?
PERIO PERIO PERIO.DIAGNOSIS,TX,GRAFTS,FLAPS,ANTIBIOTIC THERAPYBevel
incision also,
Selective grinding for occlusion..Non working and Working
PCOD,GHOST IMAGE,CEMENTOBLASTOMA..ID IMAGE
Cohort,cross sectional,clinical trial
Veracity,bene,nonmal,autonomy
HIPPA,OSHA
Partial and bridgesknow indication and how to adjust them
Noble metals..which one is a noble metal?
Orthro appliances indications1st molar and 2nd molar
Ph pka calculation
Modeling,Desentization,Behavioral..know all tht SHIT..
If I think of some more I will post it...
Pearls:
Ok my day 2: It was EXACTLY the same cases from the 09-10 which another
classmate postedso you have the casesbasically the exam writers were lazy if
they dont change anything for the next couple weeks, everyone should pass,
because its a rockand day 2 is an EAGLE ! I will say that you still need to know the

information because it seemed like it could be more than one answer for the majority
of the questions, so I wouldnt necessarily pick the same answers pick what you
think is best.basically if theyre not changing the cases, review ortho, operative,
tufts pharm, and treatment planning, And dont make the same mistake I made, I ran
out of time and left the last four questions unanswered.
I honestly dont think theyll change it next weekbut you never know.and as for
Januarywho knows?? Best luck to all of you
Day one:
Ok.sooo basically any question that I saw repeated was from the 09-10 and what
everyone else has postedso definitely look at those
I wouldnt memorize all because some of the questions are asked differentlyso just
know the topic (I realized that they were just asking the same questions differently
and maybe changing around the answer, because I would get to one and tell myself I
just saw this a couple questions before)
Public health came from somewhere in mars.i dont really know where to tell you to
look because I read mosby, dental decks, and dental secrets and didnt see the words
they were using.I guess google/ wiki the types of insurance.. HMO/PPO/ etc know
more than just the basic definition
And just in generalknow mosbys public health like the back of your handI
wouldnt put a lot of effort into memorizing sterilization stuffonly question I had was
the disinfectant definition.
Question about what would not be included in HIPAA something about PHI and what
is covered under the regulations .(ummm ??? some type of private health
information? lol Idkmaybe Im slow )
A question about RL at the furcation of a primary molar?? Necrosis, trauma, normal
anatomy, erupting premolar
A dumb question about what type of mercury is used in dentistry.ethylmercury,
methylmercury, normal mercury, .who the f cares
A lot of really nit-picky questions on porcelain and veneers.and properties
Patient just has percussion sensitivity .. no other clinical/radiographic symptoms
what is it? Periapical abscess, periodontal abscess, APP, and I think CPP was also a
choice
The sequence of extractions and whybut they only gave the max molars (so like 1 st
2nd 3rd ).and none of the answers had anything to do with protecting mandibular
sockets, etc
Oral DX: was a lot of basic path this was actually the easiest section in my
opinionPCOD twice, complex odontoma, mucus retention cystonly got like three
questions on radiology about overlapping, etc.then a few identifications .nutrient
canal, pterygomaxillary fissure, mucus cyst in sinusetc
Pharm: pretty straight forwardstraight from tufts, exact questions actuallybut it
was a good number of questions where they would give you symptoms of like

overdose or toxicity, and you had to pick the associating drugor ask you what
would you do first in certain situations of course the pregnant lady ?s bingo!
(right hip upduh)
Endo: simple stuff like lingering pain, what is it?....
Pedo/Ortho: mostly pedoknow calcification times, I had 2 questions, a whole lot of
common sense behavior stuffsome weird question about movement during
orthodonticsso just understand the changes that occur in the pdl/ bone etc
Operative, Prosth, and Perio were the ones that were really specificlike where in the
oral cavity would you do/not do certain flaps and why, specific measurements for
crown preparations (veneer, all ceramic....I dont remember seeing anything on
goldwouldnt waste my time memorizing that ish..just basics).
-Operativeknow composites cold
most questions were specifics on like the results from water:powder ratios,
overtrituration , what causes expansion of different materials
one question about you do an MOD and the patient experiences sensitivity to cold or
somethingyou take out the occlusal portion and redo it, that relieves the
symptoms, so what was the sensitivity caused from? Answers were worded strange:
something about gap causing bacteria to get into dentin tubules, cant remember the
rest
-couple simple implant questions.the one about 2 stage implant is mobileI chose
it failed so remove it
Overall, its fairthe problem is they speak in 1800s slangso its hard trying to
figure out what theyre asking they use weird terminologythey wont give you
words or drugs youve seenbut their made-up words will have that word in it.
For example, they asked what drug is used to treat Herpes, Chickenpox, and CMV,
answer choices were Amantadine, a fungal one, a retrovirus drug, and then
paracyclovir?? What? I suppose thats acyclovirso thats what I chose.
Splash:
If you still have time, do those Kaplan q-bank questions. They were the truth.com
Eye-related accidents at the office- most often affect: Dentist, hygienist, dental
assistant, custodial worker
Puncture wounds at the office come from: during the procedure, clean-up, re-capping
the needle
Most common impacted tooth- mand 1st pm., max 2nd pm, max canine, mand canine
Access opening in maxillary pm, which wall is most likely to be perforated? Mesial,
distal, palatal, buccal
Xray: identify u shaped radiopacity around max 1st molar: zygomatic process
Best xray to view zygomatic handle: Pan, Occlusal, Waters, one more (submentovertex wasnt a choice)

Mechanism of Action of Carvidopa- why is is good to use for Parkinsonism


All of the following cause gingival hyperplasia except: answer was Acyclovir
The role of microbial plaque is most obscure in which of the following diseases:
gingivitis, periodontitis, desquamative gingivitis (what I chose) ,Necrotizing ulcerative
gingivitis
Most of government funds for dental treatment come from: Medicare, Medicaid, HMO,
grants
Immunofluorescence and IgG is used to diagnose: Pemphigoid, Erethema
Multiforme.
How to treat ANUG
Porosity is most often seen in porcelain when..Firing temperature is too high
Stridor is diagnosed as bronchospasm, laryngospasm
Patient in dental chair and starts expiratory wheezingprobably due toasthma,
copd,
Most common dental emergencysyncope
Coronary artery disease in children linked to.obesity
Population of 1000. Last year 200 cases of disease. 1 year later300. Whats the
incidence? 100/1000, 400/1000etc
Max. dose of lidocaine you can give to a 3 year old that weighs 16 kg? 10 kg, 54, 72,
115
24 month old presents to the dentist with 14 teeth with caries? Whats the best way
to treat? Premedicate and local anesthesia, nitrous oxide and local anesthesia,
general anesthesia, local anesthesia and restraint
Xray with description: 19 year old, scalloped radiolucency, teeth are vital? Traumatic
bone cyst
Decreased alkaline phosphatase and premature exfoliation of primary teeth?
Hypothroisim, hypoparathyroism, hypophospatasia
What is associated with osteomas? Cleidocranial dysplasia, gardeners
What malignant salivary glad tumor has best long term prognosis? Polymorphous low
grade, mixed malignant, adenocystic.
How to treat a ranula? Excise the top of the lesion, excise the submandibular gland,
aspirate
Mucous retention cyst most commonly locatedlower lip, floor of mouth
Oral lesions of tuberculosis..

Lesions in mucormycosis resembleulcers, swelling of the tonsils


Patient has endo/perio lesion..how do you treat? Endo first, possible perio; perio
treatment only, perio treatment then endo
Couple questions on prevalence, incidene, etc
Freud and somebody else say that Anxiety is. (definition of anxiety) ie. Normal
behavior, needs to be suppressed for proper maturation
Lil boy ( I think he was 7) has severed decay of primary mandibular 2nd molarsdistal
shoe, bilateral band and loop, lower lingual arch
Piagets theory of.
What appliance causes maxillary advancement- high pull head gear, cervical pull,
protraction head piece
First symptom of cavernous sinus thrombosis- loss of vision, swelling of the eye
Highest population of oral cancer- black males
Purpose of acid etching
Porcelain is strongest under compressive forces
Antidote for benzodiazepines- flumazenil
Morphine affects what receptors: Mu, Delta
Opiod overdose.
Patient has swelling of lower left eye lid and can not feel the skin on the left side of
the face..what is fractured? Roof of the orbit, floor of the maxillary sinus, medial wall
of the maxillary sinus..
Endo diagnosis
Most important when treating the elderly- speak slowly and clearly, involve them in
the decision making process,.
Alkylating anticancer drugsMOA
Place a composite..its too light. What do u do? Replace, composite tint
Patient has shrunken face appearance.what needs to be done? Increase VDO,
Decrease interocclusal distance, etc
Purpose of the facebow
Disadvantage of Polyether
Cementation of PFM. What do you check first? Margins, contour, esthetics,
Interproximal contact

Best drug to give for HIV patient with oral candidiasis? Systemic fluconazole, Topical
something
Nitrous Oxide is contraindicated in pregnant women. When is best time to
administer? 1st tri, 2nd tri, 3rd tri, No tri
Most common symptom/ side affect of Nitrous Oxide- muscle relation, bronchospasm,
bradycardia..VOmitting or nausea was not a choice
Know when PTT, PT is prolonged, shortened (inder what circumstances)
Antidote for Acetominophen
Aspirin is contraindicated with what? Benzodiazepines, Tricyclic antidepressants, 2
other choices
Patient has wear face on the mesial incline of the mesiolingual cusp of max. 2nd
molarwhere else can u expect to find wear on the mandibledistal incline on
distobuccal cusp, mesial incline on db cusp, mesial incline on mesiobuccal cusp,
distal incline on the distobuccal cust
Best way to detect a fracture tooth? Xray, visually, Have pt. bite down locally
If you want to reduce pocket depth and not
Titanium is best used for implants due to its.biocompatibility
Absolute temperature in bone before necrosis after 1-5 minutes? 51, 55, 43, 47
Patient gets some kind of infection after administering an IANB? What space is it
most likely inpterygoid, cant remember the others
Whitened a tooth, best time to place composite restoration so that you dont mess up
the bond or something like that? 1 hour after, 24 hours, 3 days, 1 week
Most common complication with restoring implants? Mobility of the implant,
loosening of the screw
Type of tissue found between bone and implant? Type 1 collagen, sharpeys fibers,
fibronectin
People who truly have the disease- sensitivity
Know how to treat fear and anxiety
Need for caries- plaque, bacteria, host..
Patient is taking hydrochlorothiazidewhat test do you need to take? PT, PTT,
electrolyte
Patient in the chair and feels illwhat do you do? Lay them in trendelenburg,
administer oxygen

Pregnant woman 8.5 preggers and feels light headed. What do you do? Turn her
slightly to the left, lay her back even more
Best way to prevent osteoradionecrosis,pretreat with hyperbaric oxygen, extract
hopeless teeth in field radiation, extract the teeth with no precaution
Treatment planning sequence
Tetracycline stains what? Enamel, Pulp, Dentin, Cementum
Biologic Width- 1mm, 2mm, 3mm,4mm
Gingival margin on Tooth #29 is 1 mm below the cej, pocket depth is 3mm. whats the
attachment loss?
Most important factor for retention of crown? Axial taper, retention grooves,
Fixed resin restoration is at least 30degrees, 180, 270
What best benefits an edentulous patient.removable dentures, over dentures
supported, fixed implant prosthesis
Patient asks you to change a date on insurance claim. That isFRAUD
U tell patient they need to change their amalgam fillings to compositewhat ethics
principal are you violating? Veracity
Patient has right to self governance..Autonomy
Adverse effects of glucocorticoids
MOA or adverse effects of Sulfonyureascant remember what the question was
Prophylaxis of Angina- propranolol, digoxin.
Propanaolol MOA.blocks beta adrenergic receptors
Patient has a denture. Complaining of burning sensation of lower lip..compression
of the mental nerve
Radiolucent lesion around the pericoronal tooth..Dentigerous cyst
Giant cells are associated withhypothyroidism, hyperparathyroidism.
AcantholysisPemphigoid, Pemphigus, EM,
Why the hell do you bevel the max. occlusal rims? 4 long answer choices
Prevalence of cleft lip/palate
Cleft lip palate- most affected by age, genetics,..
Pan- had to identify soft palate
Lidocaine vs epinephrine effectswhat casuses what

Discoloration of a recently traumatized lower incisor is due to what? Chromogenic


bacteria, blood in the dentin, internal resorption
Replacement resorption- know causes and characteristics of it
Maxillary advancement- lefort 1, distraction osteogenesis, bilateral sagittal split
What muscle is involved in facorable fracture? Temporal, masseter, mentalis..
Emergence profile of an implant- 2-3 mm apical to CEJ
Bone graft with most predictable outcome? Auto, Allo, Xeno
Palatal tori removal when fabricating a dentureknow when and when not to do it
Patients ridgeundercuts on tuberosities, for denture fabrication. What do you do?
Nothing, reshape one for favorable insertion, remove undercuts on both
Why do you do sinus lift? Something about implant placement
Indications for 3rd molar extractions
Indications for use of calcium hydroxide
Sodium hypochloriteall of the following are properties ofexcept; Ans: chelating
agent
ANtiobiotic Prophylaxis: Mitral Valve Prolapse WITH regurgitation
Antibiotic prophylaxis: Prosthetic heart valves
Intercuspal distance is greater than 1/3. What type of restoration? Amalgam, onlay,
inlay, crown
Reducing cusp is used for Retention form, resistance form, convenience
Carbide bur with many bladesincrease efficiency and smoothness.all those
different options
What type of bur gives smoothest surface: cross- cut, fissure bur, diamond
When do you restore a tooth? When still in enamel, when it reaches the dej,
What will not get rid of interproximal plaque? Proxy brush, tooth pick, floss,
toothbrush
Dentists most likely see what clinically in bulemics? Enamel erosion.
Gastric acid most likely causes what? Abrasion, abfraction, erosion, attrition
Specifics about Pin placement- 1.5 mm axial to dej, all kinds of choices. (Refer to
mosbys)
Advantage of Glass Ionomer.release fluoride
Angulation of the blade for SRP

Dentic sensitivityhydrodynamic theory


Most common place for caries? Apical to contact, coronal to contact, fissures of
occlusal surfaces
Worst prognosis? Perforation at apex, 3mm apical to apex, 3 mm coronal to gingival
margin, 3 mm apical to gingival margin
Recently intruded permanent tooth. First step in treatment? Splint, rct, reposition,
nothing
7year old. Receives .2ppm of fluoride? How much do you supplement? 1 mg, .5, .75,.
.25
Acidulated fluoride concentration? 1.23%, 5%....
When should patient sign informed consent? During treatment planning.
Material most likely to abrade natural dentition: gold, amalgam, composite, porcelain
Contraidications for implant placement: cardiovasculardisease, smoking, radiation
therapy
Dentist agrees to providing serices to a certain group of people for certain fee. Funds
run out. You can now, charge your regular fee, work out agreement with patient,
honor fees
What affects the posterior palatal seal? Vibrating line, hamular notches, (it was a list
of things for each answer choice)
Day2 :
I had every case from 09-10. Every question was asked, Im just going to add on the
extra questions I had to each case or add the reworded version.
Case 1
Long face, female, does not want to have those spaces between teeth needs to close them
(she does not have Lateral Incisors) Good oral hygiene.
1). What treatment for flare out of Central Incisors? a). With removable dentures b). Braces c).
Face-gares
Case 2
AA man in his 40s. not currently taking any medications, or has allergies to anything. Doesnt
receive regular medical or dental health.I forget the CC. But under patient info it states: Patient
didnt get regular treatment. Now that he has insurance he can afford to get regular treatment. He
wants to get his teeth fixed because he wants to keep all teeth that are worth saving. Tells the
dentist that he has some mild anxiety towards going to the clinic. Had an emergency rct on #18 2
months earlier that was completed successfully.
AA man 46 yrs afraid of dentist, #3 and #6 crown decayed completely lost.
1). Low blood pressure, low pulse - Syncope when have anesthesia
-The vitals of the patient show very low blood pressure and pulse. what is most likely
to happen? A) cardiovascular collapse, B) Thyroid storm C) Syncope D)some else
that wasnt right.
2). Fix #8,#9 esthetic edge/yellow stain. What treatment to #8, #9? (little bit cheeped off enamel
of #8) a). polished to smooth b). composite c). bleach

- Fix #8,9 the patient doesnt like how the two front teeth arent on the same level. What would be
the best treatment? A). place a composite restoration on both teeth to put them on the same
level. B). Place PFM crowns on both teeth. C). Smoothen out the distal portion of the incisal
edge of number 8. D). Place a porcelain veener on the both 8 and 9. The thing about this
question is that the distal edge of number 8 does look off angle, but the problem is that the chip is
on the mesial of 8 not the distal. (its a really small chip too). #8 does look a little off colored on
the distal facial side but they didnt ask about what to do about it.
3). What treatment for #3, #6 space? a). bridge #2-6
- Same question as above, patient also has number #4 and does have decay.
-how would you treatment plan #4 if it were to get an RCT/ what is the most acceptable
treatment plan after the RCT? Take a look at the x-rays and youll see decay at/near the
level of the alveolar bone. Choices were A) post and core B) amalgam core C) post and
crown and crown D) Crown lengthening, post and core, and crowing
-Patient tells you that his anxiety started only in his adult years. Hes 40 something now.
What could be the cause of the anxiety? A) how the dentist approached the patient B) how
the front office staff treated him C) His past dental experiences D) the cost of getting
dental work done E) people at work telling him their experiences in the dental office
-Look at #31 select what you see A) large buccal lesion B) Occlusal decay C) APP D)
something else
4). Caries small/occlusal (brown spots on pits) on #31 can be seen on a). picture clinically b).
xray c). from chart
5). #12 has big caries lesion on mesial up to the bone level. What treatment needed? a). crown
lengthening b). post&core next step because crown only can be on sound tooth structure
(ferrule rule = 1mm circular in prep increase strength in 10? Times - check this)
Case 3
I think that case was about some man that used to smoke and something about him trying to quit
smoking and trying the nicotine patches and other stuff and it not working. Hes only smokes 4-5
cigs a day.
- The best treatment to try to get this patient to stop smoking would be? A) some weird drug B)
Nicotine cream C) Nicotine nasal spray D) behavior counseling E)varenciline ( which is a anti
smoking drugs but works on the nicotine receptors too )
63 yrs old man Picture teeth radiograph
1). #14 bone lost mesialy, overhang amalgam. Why? a). forget to put wedge
-what would be the most likely reason to of the amalgam overhang and the cause of the light
contact between teeth to teeth? A) using weak amalgam, B)something dumb C) a wedge was not
placed when the tooth was restored
2). From #20 big amalgam restoration you can see on xray pin goes out of tooth distally. Patient
complained that floss shred all the time between #20 and #19. What should you do? a). explain
patient current situation b). tell pt. that previous dentist performance as bellow standard of
dentistry c). extract tooth #20 d). try to cut pin with hand piece
3). Can see well defined circular radiolucency under root tips of # 30, #31, #32. Diagnosis? a). PA
cyst b). OKC c). ameloblastoma
-same question as above. What seems to be the most likely reason of the radiolucency apical to
tooth 31? A) PA cyst B) Odontogenic cyst C) ameloblastoma D) normal anatomy ok so for this
one you need to take a look at the other side of the Pan , and if you do you can see that this RL is
in the same location and just doesnt look anything of the pathology mentioned. it looks just like

this! I put normal anatomy cus its the submandibular gland fossa. PS in the Pan on the exam

you can also clearly see the mental foramen on both sides.
Case 4
53 yrs old Porcelain-fused-to-metal PFM bridge #8-10. On PAN can see 3rd molars are impacted.
1). Why discoloration of bridge white color/translucency. Every explanation is possible EXECPT?
a). metal to thick b). not enough reduction in cervical third of #10 c). opack layer is too much
thick
-same question, it asked why the discoloration of the cervical third of the #10. A)not enough labial
reduction b)opaque layer too thick C)metal too thick D) something else . Anyways in the clinical
picture you can see that the cervical third near the gingiva is clearly more white than the rest of
the crown.
2). On clinical picture you can see wear off mandibular incisors. What is a reason for that? a).
occlusional habite (bruxism) b). thin dentin/enamel 3). Opposing bridge (reason for that in the
next question, if its just opposite bridge why all canines are flat?)

-whats the reason for shape of the lower incisor insical thirds? A) thin enamel B) erosion C) oral
habit D) something else

3). Why is that shape of canine no cusp, flat occlusialy? a). bruxism
-whats the reason for the shape of the canine?in the picture the cusp tip is kinda cut in
half. I dunno that looked weird to me but its the same idea I went with Bruxism other
answer choices didnt make sense.
4). Should we do 3rd molar extraction for the reason that #1 is close to sinus or #32 is close to
mandibular canal? NO (53yrs, 3rd molars are not bothering him)
5). 3rd molars are #1 disto-bucal and #32 is horizontal angulation of impaction. Plus partial bone
coverage. (check in book impaction angulations) TRUE
-some question about a tooth number 3 needing to be extracted and why separate the
tooth when extracting it.
-#3 needs to be extracted, because of its close proximity to the sinus , it is always
indicated to separate teeth It was a 2 part true or false question. the tooth was very
baldly decayed and there was no clinical crown. Something similar to this minus the
anatomical crown on the mesial.

6). What would be the reason to extract #1? a). to place implant, if #2 in future would be lost
and pt. need a bridge.
7). If you do pulpal thermal test on his posterior teeth you may have Negative/False result. Why?
a). Age b). pulpal obliteration/ calcification see xray
Case 5
On clinical picture you can see adult complete dentition (no missing teeth) in position central
incisors touching edge-to-edge. On back, posterior teeth disarticulated.
1). Why discolored pre-molar? Amalgam stain
2). What movement of condyle in TMJ must be for that position? a). rotation b). translation c).
both - for protrusive you do both
-what movement of the condyle is going on in the picture with the lady biting edge to edge? A)
both condyles are rotating ( how is that possible? Lol) B) the right condyle rotating while the left
translates C) the left condyle rotates while the right condyle translates D) both condyles on
translating.
-what kind of position if the patient demonstrating in the edge to edge picture? A) maximum
intercuspation B) Centric relation C) Incisor guidance D) Centric position.
3). What clinical picture is demonstrating? a). free way space [the space between the max and
mand occl surfaces when at physiological rest] b). maximum intercuspation c). central occlusion
d).incisor guidance (I dont know the answer I put a but may be b or d also, check it)
4). On xray radiograph you may see circular radiolucency on middle root (close to apex) on #9.
Asymptomatic, no pain. Diagnosis? a). lateral periodonal cyst b). radicular cyst c). medial
palatine cyst
5). What is the main test needed to be done for diagnosis? a). Thermal vitality test b). EPT c).
percussion
Case 6
68 year old female wanting to get some work done. Shes taking certain medication but you have
to realize that Fosamax is a bisphosphonate! This case mainly tests you on the principles of what
you can and cant you to people taking bisphophnates or have an increased risk of ORN.They
might switch out the drug name you become familiar with bisphosphonates
68 yrs female came for your appointment with old dentures (both max/mand), that didnt fit her
anymore. She had history of using Fosamax medication (biphosphonate drug to protect bones).
She is after cancer surgery, radiation, chemo therapy On xray all teeth are missing
except#6,7,8,9,10 and 25,26,27
1). What is possible diagnosis for her psychotic condition? Depression
-what is the most probable condition that this patient by have? A) Bipolar B) Anxiety
C) Depression
-if this patient is on bisphosphonates, which of the following treatments can you
render without increased risk of systemic complications A) extraction of all hopeless
teeth B) Scale and root planing C) something that you shouldnt be doing D) RCT
-An expect question? I forget the other choices but the except was that the patient
can proceed to get her extractions done after being off Fosamax for 1 week

2). If she is after breast cancer chem./radiation + biphosfonate drug Fosamax, what treatment for
her you CAN do if needed? a). extraction b). root canal c). alveolar plasty/surgery d). implants
(you cant touch bone risk of osteonecrosis)
3). After Fosamax was stoped for 1 week can you do extraction? NO, Fasle
4). What treatment is good for her? Root planning + cleaning, prophy
-what is the most ideal treatment for this patient? Choices were between A)coronal
scaling and removable dentures for both max and mand. B) coronal scaling and
implant placement in edentulous areas C) Scaling and root planing and something
elseI went with this because the patient had moderate to severe bone loss and
needing not only coronal and root scaling but also needed root planing on the
exposed root surfaces.
Case 7
Kid 5 yrs. 9 month fall 3 month ago. Tooth #F fall down. You can see on clinical picture new
erupting tooth is appeared. She has a FISTULAR, bump above #E.
1). Tooth #E has luxation. What treatment? Extraction
2). Does age of patient is identical for dental age? Yes, pt.s age = dental age
3). What would be a treatment? Sealant on all permanent 1st molars
Prophylaxis, fluoride
4). What would be a treatment for posterior crossbite? Bilateral expansion
5). On biteweens you can see small insipient proximal caries on mesial of #19 (between #K and
#19) What is a treatment? a). composite b). dont do anything c). disk between teeth
Case 8
This was a Mexican lady that has TMJ problems; I believe she was allergic to penicillin
to look out for the premedication question. she cant speak English well and has a loss
of hearing from an accident. In my case the lady did have posterior teeth messing in
the in her clinical picture. and there was a picture of the her biting edge to edge .
Mexican female. Has deafness because of accident. Parents help her in transportation and
financially. She complain in TMJ pain
1). What would be the easiest to improve? a). OHI oral hygiene b). financial limitation c).
deafness because of accident d). pain from TMJ
-the question was stated differently than above, Mine asked all of these would complicate the
treatment of this patient except, A)poor oral care, B)financial limitations C) English problems and
deafness of patient D) Pain from TMJ
2). On Xray radiograph you can see #21 is good, normal angulation, no carries, #19 distaly
tipped, a big carries lesion, # 14 is supererrupting. What treatment would you recommend? a).
build up #19 carries b). build up #21
3). What is next treatment after that? Build up with post and core in only ONE root canal True
or False (I dont know, check)
-when having to build up the molar with you put the coret matrial in one canal the mesial canal
(false). Doing this makes the canal stronger (double false)
3). What is LEAST possible when you are upringting #19? a). roots of #19 move facialy b).
encorrage of anterior teeth or #21
-what is the least possible movement when up righting #18? A) roots move forward B) anchorage
will come from the premolar and anterior teeth C) tooth can extrude and cause occlusal
interference
4). When you are upringting #19 what if possible to happend? Occlusial interfearance
5). What is LEAST possible treatment for supererrupting #14? a). Intrusion b). crown c). RCT d).
caries txn
6). If you do EXTRUSION of tooth #13? crown-to-root ratio increase and prognosis decrease
Case 9
A little girl with CLEFT on clinical picture of Maxillary you can see all teeth lined up normally in
ONE line, except #6 & #7, also #10 & #11 are parallel to each other (one behind other).
1). What is reason for strange position of laterals #7 and #10? CLEFT

2). On Cephalometric picture what is LEAST possible diagnosis? a) maxillary prognatism b).
class1 c).class2 div 2 d). class3
3). What arrow point on xray? HYOID
Case 10
Man 46 yrs also with CLEFT palate, fixed when he was a kid, by surgery. On clinical picture he
has Angular Chelulitis on corner of his mouth. Hes complaining that his dentures are moving and
discomfort him and lesion in corner bother him.
1). What is treatment for Angular Chelulitis? Clotrimasol cream 2%
-the red inflammation that has formed under the maxillary denture is because of a ? A) bacteria B)
fungus C) protozoan D) something else
2). On PAN two opacity left/right under his mandible? HYOID
3). He is missing #7 and #10 and bone here (because of cleft) look like resorbed up to10 mm.
What would you recommend treatment? a). extract #8, #9 and do bridge #6-11 b). saving #8,
#9 (not extraction) to preserve a bone/alveolar ridge (not sure I choose b)

4). Implants for #7 and


I had a case where I needed to identify the hyoid bone on a pan then find it in the lateral ceph
they provided. This is how it looked. Its basically under the angle of the mandible in a lateral
ceph. Dont get clowned I thought for a second it might be the clavicle lol The other arrows
pointed near the vertebrae to confuse you!!

Sizzle:
Lots of questions were from the 09-10 at least the first 50 pages of the 09-10
document was asked. The other load of questions were from KAPLAN!
PLEASE IF YOU HAVE TIME DO THEM.com(I dont understand this joke and it
sounds stupid)
x-rays were basic: they showed you were to look and gave you the
description of what you were supposed to identify. Same goes for pictures. I
had stuff on white sponge nevus. and one I had to identify that if the patient
had Bells palsy, ( they described it in the question and also all you need to
remember is that it looks like somebody trying to make a retarded whink!

Perio:
Flaps on flaps on flaps. Know these very well, they arent too hard but if you
just breezed over them you might have a hard time.
Know the signs of inflammation and stuff like that and you should be fine.
Pedo:
Calc. Times, and how to treat annoying ass kids. Also some questions on
eruption times of course. This section wasnt too bad. The only hard part was
the management of the kids questions other than that it was ok.
Pharm:
Basic questions!! This is what you need to look at Kaplan qbank for!! Most of
these questions were repeated from there. Had questions as basic as: why
doesnt LA work if patient has infection, INR is used for what (some options
were, to test for penicillin or some other sort of anti bacterial lol), hardest
one that I can remember I got was about the law of mass something I
dunno its in tufts pharm, and the answer is the only one that makes sense
though. If youve studied tufts Im sure youll be fine.
OS:
This section was a little bit harder that the rest. just remember to look over
facial spaces, and extraction sequence and the reasons why you extract them
that way. The 09-10 explanation wasnt what/how they asked me. Now
about ORN and when you extract that tooth (pretty straight forward). Most
commonly missing tooth, most impacted, and also least likely to be missing,
and which would be the most detrimental if it was missing ( loss of arch
length type question)
Operative:
Lots of questions on composites and the indications of them and whatnot
Stuff on indications of inlays and onlays (not too specific )
The retention factor of class 1s,2s
Had questions on the C factor
Removable:
Wasnt really hit with material type questions just try to read over fixed from
Mosbys and youll do fine in that section.
Lots of denture questions, post. Palatal seal and why its used (obviously not
for the mandibular denture like 2 of my options had)
RPD questions werent that bad either, they asked stuff such as what
connects to occlusal rest to the major connector.
Know stuff about the position of teeth and they sounds they make, asked
about this 2 times.
Endo:
Very very very basic questions. Know the different Dx. They give you choices
that are obviously not the correct answers. (whens the last time you heard an
incipient lesion causing a pulpal necrosis lol) if youve reviewed tufts endo file
Im sure youll do fine.

Pt. MGNT:
The only difficult section in this section were the stupid PPO HMO questions
they asked!! And they ask them using definitions and examples not used in
Mosbys. Use a different source other than mosby for that part.
If I were to do anything differently in my last week of studying, I would have
read the 09-10 front to back and also read Mosbys again.
Questions I got: again I think a lot of mine were from 09-10 so you might
want to look at that before your test date.
Best way to view maxillary sinus? Waters view
Question about LED lights, it was an except questionyes they can be
powered by battery
How can you tell if the infection is of non-odontogenic origin?
Referred pain question. Lower mandibular molar was the answer

23. Pregnant lady? Lay right side up, what is artery are you
protecting?
Inferior vena cava
26. Mandibular incisor coming in crowded how do you make space?
Interarch distance from primitive space
Neurapraxia, the least severe form of peripheral nerve injury, is a
contusion of a nerve in which continuity of the epineurial sheath and
the axons is maintained

32. 5 year old child having pain what do you give them? Asprin,
ibuprofen, codeine, acetominphen
36. OKC-most likely to reoccur
37. Nevoid BC
Nevoid basal cell carcinoma syndrome (Gorlin syndrome) is an
autosomal dominant inherited condition that exhibits high penetrance
and variable expressivity. The syndrome is caused by mutations in
patched (PTCH), a tumor suppressor gene that has been mapped to
chromosome 9q22.3-q31. Approximately 35% to 50% of affected
patients represent new mutations. The chief components are multiple
basal cell carcinomas of the skin, odontogenic keratocysts,
intracranial calcification, and rib and vertebral anomalies. Many
other anomalies have been reported in these patients and probably also

represent manifestations of the syndrome. The prevalence of Gorlin


syndrome is estimated to be about 1 in 60,000.
39. Material least to do impression with-irreverisble hydrocolloid
Polyether is the worst p334 (mosby )
40. H2 histamine-gastric reflux
H2 inhibit the action of histamine in the stomach and reduce stomach
acids.
ex. Rantidine, cimetidine, famotidine.
45. Calcified canal what do you do-refer
46. When is it ok to do a temporary fixing on patient? Emergency
47. Incidence 100/1000
Incidence: indicates the number of new cases that will occur
within a population over a period of time (e.g., the incidence of
people dying of oral cancer is 10% per year in men aged 55 to 59 in
our community).
How do you get a child acting out to act favorably? Let them watch
another child behaving-Modeling technique.
54. Class 3- cleft palate, cleft lip
59. Bevel for occlusal on a crown? structural integrity
68. Amoxicillin and clonavonic acid is combined to keep from
degrading beta lactam ring
75. Why do you take denture out at night
Patients should be told that dentures must be left out of the
mouth at night to provide needed rest from the stresses they
create on the residual ridges
72. If patient wants to last for 8 hours which is long acting drug?
Aspirin, ibuprofen, acetominaphine, n-something
82. ANUG comes with spirochetes
83. Pic of white spongy nevus- the clue on the description was
plaques!!
White sponge nevus: of buccal mucosa.
The lesions of white sponge nevus usually appear at birth or in
early childhood, but sometimes the condition develops during
adolescence. Symmetrical, thickened, white, corrugated or
velvety, diffuse plaques affect the buccal mucosa bilaterally in
most instances
88. How long do you splint with avulsion. 7-10 days,
91. How much do you take off facial for veneer? .5-1m
90. Ppm in water-1

1 ppm = 1 gm/L
95. Contraindication for diazepam-diabetic, pregnancy, etc
97. Ging recession 5-6mm on #4 & 20, Hemoglobin of 12. Wht do
you do? Treat, refer to dr, scaling n root planning
Hemoglobin (male)
13.5-17.5 g/dL
Hemoglobin (female)
12.3-15.3 g/dL
98. What muscle covers denture? Buccinators, masseter, lat & med
pterygoid
99. What provides lingual retention? Mylohyoid
100. Neurofibromatosis- axiallary freckling, caf- au-late, lesch
nodules
101.
Most impacted tooth? Mx k9
108.
Base metal vs noble metal-single crown-3 unit
bridge
103.
Purpose of hex implant :
in an internal hex implant, the antirotational feature
104.
Push on rest seat it comes up? Base doesnt come up
bc of resin
105.
2nd to s. mutan-L. bacillus
109.
Papillon le fever
110.
Oligiodontia-ectodermal dysplasia
111.
Collimation-tube
A collimator is a metallic barrier with an aperture in the
middle used to reduce the size of the x-ray beam
112.
Erosion- bullemia
113.
Patient gets 25% home bleaching. Wrong its 10% but
2nd part is true
The current home bleaching technique, employing a customfit tray containing 10% carbamide peroxide solution, was first
used by Klusmier in the late 1960s.6 In-office bleaching materials
are usually supplied in concentrations of 35% hydrogen
peroxide, although some concentrations may be as high as
50%. The caustic nature of 35% to 50% hydrogen peroxide mandates
that the soft tissues be isolated from any possible contact with the
bleaching material
114.
What goes into cavernous sinus from upper lip?
Subcutaneous tissue
115.
URI-no NO2
116.
In posterior composite why do you have to redoocclusal-wear
117.
Periosteum-sharpeys fibers, cementum, alveolar
bone, or all 3

118.
Symphisis-intraocciptal, sphenoocciptal, which
bone forms last
122.
Nonworking-bull working-lubl
123.
Transillumination-vertical fracture
124.
Minor connector connects to
127.
To far superior and anterior dentures-what sounds
The labiodental sounds f and v
128.
If you did a DO what axioline angle is not there
129.
If you fall and break incisor which class is it due to?
Class 2 div 1
133.
Support area for max and mand denture
Maxilla: residual ridge primary, rugea
secondary
Mandible: buccal shelf primary
134.
Cleidocranial dysplasia-supernumary teeth
136.
Nausea and vomiting from opoid receptor poisoning?
Chemoreceptor trigger zone
138.
PCN and tetracycline cancels each other out
1.
Guy has problem with a tooth and has a hole drilled thru the O of MOD
composite and the pain is relieved. What caused it? polymerization
shrinkage
8.
Sialolith commonly found? Submandibular gland-whartons duct
6.
Neuropraxia question-nothing severed, perioneum intact, can get it from
stretching.
11. What is best way to view TMJ? MRI
27. Grand mal(tonic-clonic) seizure drug of choice? Dilantin( phenytoin)
25. What is malignant? 25. What is malignant? Fibrous dysplasia, pagents ,
central giant cell granuloma???
32. What do you do with probe if furcation is wide and narrow, narrow, wide?
Probe or cant probe? Grade 1 probe goes less than 1/3, G2 probe
goes more than 1mm(do GTR n graft), G3 probe goes straight thru
Grade I is incipient bone loss, grade II is partial bone loss (cul-de-sac),
and grade III is total bone loss with through-and-through opening of the
furcation. Grade IV is similar to grade III, but with gingival recession
exposing the furcation to view.
(Newman, Michael G.. Carranza's Clinical Periodontology, 10th Edition.
Saunders Book Company, 072006. 28.5.8).
33. What do you do for a furcation that you can see through? T or F.
Tunneling, GTR membrane?
Class I: Early Defects
Incipient or early furcation defects (class I) are amenable to
conservative periodontal therapy. Because the pocket is
suprabony and has not entered the furcation, oral hygiene,

scaling, and root planing are effective.15 Any thick overhanging


margins of restorations, facial grooves, or CEPs should be eliminated
by odontoplasty, recontouring, or replacement. The resolution of
inflammation and subsequent repair of the periodontal ligament and
bone are usually sufficient to restore periodontal health.
Class II
Once a horizontal component to the furcation has developed (class II),
therapy becomes more complicated. Shallow horizontal
involvement without significant vertical bone loss usually
responds favorably to localized flap procedures with
odontoplasty and osteoplasty. Isolated deep class II furcations
may respond to flap procedures with osteoplasty and
odontoplasty. This reduces the dome of the furcation and alters
gingival contours to facilitate the patients plaque removal.
Classes II to IV: Advanced Defects
The development of a significant horizontal component to one or more
furcations of a multirooted tooth (late class II, class III or IV 13) or the
development of a deep vertical component to the furca poses
additional problems. Nonsurgical treatment is usually ineffective
because the ability to instrument the tooth surfaces
adequately is compromised.30,36 Periodontal surgery,
endodontic therapy, and restoration of the tooth may be
required to retain the tooth.
34. 8 year old Central incisor canal is constricted but has apical RL what do
you do? Refer
35. What is worst if doing a RCT? Insufficient obturation, insufficient
cleaning and shaping,
36. In RCT was is plastic post good to use? Same strength as dentin, better
strength then steel, same strength as steel, when cemented you can view on
xray
37. RCT done and years have RL below what caused this? actinomyces
38. Xray of woman who had molar extracted, now has infection, what caused
this? Osteomyletis, residual cyst
A residual cyst is a cyst that remains after incomplete removal of the
original cyst. The term residual is used most often for a radicular cyst
that may be left behind, most commonly after extraction of a
tooth.
39. C factor(configuration factor)- composite ratio for bonded to unbounded
The C-factor is related to the cavity preparation geometry and is
represented by the ratio of bonded to nonbonded surface areas.
Residual polymerization stress increases directly with this ratio.
40. Bilateral split osteotomy what nerve do you worry about severing?
Inferior alveolar

41. Cleft palate/lip- class 3


42. Main reason for redoing anterior composite-discolored
43. Fluoride- how much do we use in community water 0.7-1.2 ppm

78. Fractured mandible how long is appropriate to keep in closed reduction?


4weeks, 6 weeks,
Winter2011
1.
Which test is MOST valuable in an tooth that needs testing that has an open
apexyoung tooth = cold test (EPT does NOT work)
2.
Internal bleaching will MOST likely cause? Extracanal cervical resorption,
replacement resorption, external resorption (idk? Was not aware of what cervical
resorption was especially when given extracanal cervical resorption AND external
resorption in answer choices???)
3.
Apexification-done when tooth is NOT vital (aka need RCT) and you need to close
the apex so you can get a seal for the gutta percha; Apexiogenesis-done when tooth IS
VITAL and RCT is NOT necessary no matter what has happened (but you have to know
the situations in which a tooth would be vital or nonvital)
4.

When tooth is traumatically intruded LET IT ERUPT!

5.
Difference between reversible and irreversible and necrotic SYMPTOMS how
long pain lingers to COLD test etc.
6.

Sensitivity to percussion and biting you know you have acute apical periodontitis

7.

MOST likely cause of pulpal inflammation= DECAY/BACTERIA/CARIES etc.

8.
Pain from which one, mandibular premolar or mandibular molar, refers to the ear?
Idk? I have a hard time choosing between the two
9.
SLOB rule question, Buccal root shot from M, now shoot from D and its oppositie
the lingual root shot from mesial blah blah blah (SAME LINGUAL OPPOSITE BUCCAL)

10. Vertical root fracture= EXTRACT


11. Disadvantage of NaOCl: toxic to tissues (does NOT remove smear layer btw)
12. Difference b/w self etch and total etch: self etch does NOT remove smear layer
13. Bacteria responsible for pulpal infection: not specifically which one but answer
choice was one or more than one bacteria? Idk
14. Ledge bypass the ledge
15. Tooth most likely to have 2 canals: max 1st pm
16. Where canals are in mand molar most likely: 3= 2 M and 1D
17. Which access preparation canals look like a C shape? I forget mand molar?
18. What facilitates RCT NOT calcified canals
19. pH 5.5 critical
20. treatment sequencing Mosby p. 38
21. problem with amalgam lies in environmental disposal
22. difference b/w resistance and retention forms
23. knowing that liners and bases and recurrent decay can all appear radiolucent
24. on a pan 2 bones that can appear below mandible= hyoids
25. place calcium hydroxide then resin glass ionomer base then restoration
26. admix= spherical and boxy particles
27. breakdown of a composite margin most likely due to: shrinkage during curing,
expansion, or overfinished margins? Idk
28. large posterior decay: large composite is NOT ideal
29. COTE = gold< amal< unfilled composite
30. Alveolar osteitis= dry socket
31. Bleeding after ext 3 hours later remove clot to locate location of problem
32. Pt has 3rd molars what do you do: tell the patient 3rd molars cause crowding and
need to come out, do not inform patient about them, 3rd molars are associated with cysts
and various pathologies and need to be extracted, or do not do anything about them

33. Pt has cirrhosis of liver: what is his liver mostly composed of? Hepatocytes,
fibroblasts, hematopoetic cells idk
34. Pan had a 3rd molar that was basically straight up and down maybe tilted to distal by
2 mm but erupted fully it was NOT distoangular impaction (know what these look like
on an xray-distoangular, mesioangular impaction)
35. Day after ext pt comes back with fever and sick feeling give him different
antibiotic (bc he has AA bacterial infection which is associated with ext is what I thought)
OR refer him to proper specialist idk
36. Adult pt has crossbite needs fixin: listed 3 ortho appliances-even quad helix (tricky),
or surgery- adult so I chose surgery
37. Distraction osteogeneis good for LARGE movements
38. ANB 6 Class 2 dental or Class 2 skeletal malformation?
39. Trigeminal neuralgia is NOT associated with a spontaneous dull ache
40. Pt reports 3rd molars have been ext years ago pan reveals small round RO in
area of #17: osteitis
41. Pt has cirrhosis of liver: what is best: lidocaine, mepivicaine, bupivicaine, articaine
42. Infected tissue LA will be in ionized form
43. Prilocaine- methemoglobinemia
44. Swelling in front of SCM:
45. Cleft lip/palate 1:700
46. First dental formation: (weird!) 6 weeks, 16 weeks, 32 weeks idk (teeth, palate,
what idk what youre talking about)
47. Syphilis lesion that looks most like herpes? Idk
48. Pemphigoid against basement membrane (pemphigus against something else)
49. PCOD ant aa female ANT MANDIBLE
50. Xray teeth with no pulps: DI pg 124 mosby
51. Target made of tungsten
52. Mosby 141 post wall of zygomatic process on xray
53. Man w/ ill fitting partialsindurated ulcerated lesion lat border of tongue: SCC or
trauma from partial? SCC and biopsy

54. Untreated decay mostly in AA oral cancer mostly in AA males mosby 207
55. Pg 163, 164, 165 know like back of your hand quad helix corrects crossbites, use
these all in GROWING ppl, surgery for adultwhen to use LLA or band loop PLEASE
KNOW THIS! LLA-bilateral loss, #19 and 30 NOT erupted yet= need distal shoe if they
are erupted need BL
56. Chi square= categories, ttest=averages or means
57. Modeling pg 225 mosbys
58. Probing + recession= clinical attachment level
59. Do NOT attempt perio surgery until you have tried and failed at initial SRP therapy
60. 45-90 angle on SRP instruments
61. Most cost effective: stress oral hygiene home care
62. Pg 272 intrinsic activity and maximal effect and efficacy and receptors and affinity
just know it all and how its all interrelated
63. Beta blockers end in olol, anti GERD drugs prazoles
(omniprazole=tagament/prevacid etc)
64. Pharm: carbamazipene, atropine, mechanism of tricyclic antidepressants
65. If you change vertical dimension occlusion during fabricating a complete denture
what do you have to REDO? CR or facebow CR!
66. How to help pt who gags with their denture? Tell em to put the denture in for as long
as they can, put a spoon and hold it for as long as they can tolerate it-do this over and
over-YES
67. Implants 3mm apart
68. 323-324 mosby denture phonetics just know why you make all the sounds ALL of
them, esp f,v, ph (do it yourself and youll remember)
69. Kennedy classifications must know cold
70. Value is lightness or darkness
71. Primary tooth with most buccal and lingual convergenceidk
72. Hemoglobin type in sickle cell disease: A, C, F, or S? idk S
73. Pan with laterals missing in photograph but present on pan just impacted, 9 years
old, canines impacted (#1 dental age conincides with his chronological age-NO #2
dental anomaly in this kid occurs more in permanent than primary dentition, this occurs

in proliferation stage-TRUE OR FALSE idk)


74. Pan with 2, 14, 15 w/ 50% bone loss and bombed out 14 and 2 and unilateral
edentulous area on mand tx plan? A.crown lengthening and crowns B.orthodontic tx
on maxillary C.max class 2 partial D. MAND CLASS 2 PARTIAL
75. Radiolucency on pm with MO amalgam could be all EXCEPT: recurrent decay, liner,
base, something off the wall
76. your fees are high! your response should be? it seems you are concerned if
getting your dental care is worth it? OR my fees are the same as all the other dentists
or I have to pay bills blah blah
77. Modeling use sister or other kid to show uncooperative kid what to do
78. Reason for everything: PLAQUE
LaJoy

Well very interesting is all I can say about this test. From what I was
given, the only advice I can offer is dat Remembered Questions are
GOLDEN.com!!! lol!
Make sure you do as many as you can find. And try to look up the
answers to the ones that dont have definitive answers. The 09-10 is
helpful also if you start on it about two weeks before your test and get
through it all. Then the last week just do remembered questions
remembered questionsand then do some more remembered
questions! Lolol
I tried my best to remember as many new questions or variations of
the questions that have already been posted as I could. The ans
choices are either the ones I kept going back and forth b2wn or the
only ones I could remember so double check the validity..lol GOOD
LUCK CLASSMATES!
1. Mandibular division of the trigeminal nerve exits the skull from what
foramen? Foramen ovale, Superior Orbital fissure
2. How is fluorosis classified? Amount ingested, #of surfaces fluorosis
divided by the # of total teeth, # of teeth with fluorosis compared with
the # not
3. Effective dose vs. Absorbed dose
4. Distance is 12ft instead of 4ft. Calculate the distance from the source
5. Clinical attachment lost is? CEJ to the bottom of the pocket, gingival
margin to the bottom of the pocket
6. Metal denture base (Which is not an advantage)? Thermal conductivity,
weight on the maxillary arch
7. What does IgG do? Bind to the host antigen, affect IgE and mast cells
8. All the walls are missing except the distal wall which is present. What
type of defect is it?
9. Distal wedge

10.Gingivitis in puberty and pregnancy caused by? Plaque, estradiol (


most potent for sexual maturity ), estrogen, or progesterone
11.What structure is crushed when preg. Woman is in the SUPINE
position? IVC, abdominal aorta
12.Picture of bump on the gums and pt is NOT pregnant: Parulis,
Pyogenic Granuloma, Peripheral Giant Cell Granulom
(parulis: subperiosteal abcess of the gum)
13.What has histology similar to congenital epulis: granular cell tumor,
peripheral ossifying fibroma
14.Which has the characteristic to have carcinoma in situ? Leukoplakia,
Erythroplakia
15.Where do you see increase in alkaline phosphatase?
Hyperparathyroidism
16.Punched out radiouluncy? Multiple myloma
17.RL in the midline of the palate of vital teeth b2wn the max CI:
nasopalatine duct, median palatal cyst
18.A pic of a RL around an impacted 2nd Mand PM and the primary 2nd
Molar sitting on top of it? Dentigerous cyst
19. A lesion that malignancy is suspected wht should you do? Needle
aspirate, incisional biopsy
20.Pic of a erythemous lesion on the middle of the tongue and description
says it is also on the palate? Syphilis, gonorrhea
21.Bilateral red erythemous lesion? Erosive lichen planus
22.Disease to know/look up: Sturge Weber, Lymphangioma, Fibroma,
Herpangia, Actinic Keratosis
23.Which would you not see a radiographic difference? Dentin dysplasia,
enamel hypoplasia, AI
24.Pic of gingival hyperplasia? Fibroma, Fibromatosis
25.Slaviary gland obstruction of the partiod (Stendons) gland?
26.Which of the following is the last sense to leave after LA
administration: pain, temp, touch, pressure
27.Osteoradionecrosis of the jaw is likely seen in? 2300 rads of radiation
therapy, IV bisphonates for a year, chemotherapy
28.The speno-occiptal synchondrosis is: a suture, cartilage,
intramembraneous
29.Govr regulation in 1997 that mandated? OSHA, HIPPA
30.Dentist file insurance for post and core buildup and crown. Insurance
says that they should be charged 2getha and only reimburses for the
crown. What is this an ex of? Downcoding, upcoding, bundling,
unbundling
31.Why is polyether not a good impression material (disadvantages)? Get
stuck in the pt mouth, low solubility
32.Dentist is doing a MOD composite restoration. Pt complains of pain 3
days after. What is the reason? (polymerization shrinkage was NOT an
answer) pressure from expansion
33.Advantage of indirect comp?
(advantages of indirect comp frm Kaplan: less polymerization
shrinkage, better proximal contact, less marginal leakage, greater
strength, less post-op pain)
34.Which drug is NOT used to tx Angina? Nitrates and nitrites,

35.Which drug does NOT play a role in platelet funx? IB profen, Asprin,
Ginseng, Plavix
36.Digoxin assoc with kidney funx?
37.Which is true of pseudomembranous colitis? Due to overgrowth of
clostridium difficle, drug of choice to tx it is metronizadole

As far as the case questions, they werent so bad. Acutally kindof fun
made you feel like a forreal dentist (lol!) which is also a bit scary b/c
wht you think is a good tx ADA might not agree. So the best thing I can
say abt that is go over your orthohad A LOT of ortho (bimax
protrusion, class I, Class II, profiles, if the ANB angle is 6 what type of
profile does this pt have, and lateral ceph radiographs). A couple of
Endo diagnosis, and a lot of prophylaxis, like should pt be
premedicated n if so wht to give (so knw reasons for premedication
endocarditis and total joint replacement..and the dosage)
Jeweled
Hey folks!!! If you haven't opened up the 09-10 file OPEN IT!!!! LEARN IT!!!! KNOW IT!!!! But most
importantly UNDERSTAND IT!!!!!! If I were you I would also look at any other files related to 09-10.
My test had a lot of ENDO, it wasn't hard but just be clear abt ACUTE PERIRADICULAR ABSCESS vs
ACUTE PERIODONTAL PERIODONTITIS (I think I wrote those correctly...)
I had about 5 hue, chroma, value questions...I can't remember the questions but if I do I'll post again.
Oral surgery:
What is the ext sequence for molar ext 321 to protect the tuberosity, 123 to protect the tuberosity, 213, 321
etc, etc
What is the most common complication after ext? dry socket, infx, hemmorhage, something else can't
remember
what is a complication with bilateral split osteotomy? Damage to the IAN.
If a pt has been taking corticosteroids for a long time what do you have to be concerned with? Adrenal
insufficiency
If a pt has been taking bisphosphonates they may be at risk for osteonecrosis b/c: they have to have
radiation therapy (something like that)
If a pt is going to have head and neck radiation what should be done? the pt should have questionable teeth
extracted prior to radiation
Fluoride:
SO THAT YOU WON'T GET CONFUSED I WOULD RIGHT DOWN THE SUPPLEMENTATION
CHART BEFORE YOU START THE TEST.
What is the supplementation for a 5yr old in a community with 0.28ppm fluoride?
What is the supplementation for a 7yr old(??) if the community water is 0.75ppm?
Pedo:
When is calcification for a max central incisor?
Ortho:
If a pt has had MENARCHE (her PERIOD aka menstral cycle) what does this say about her growth spurt? I
put it is AT the PEAK of the growth spurt ( the other choices were before or after the growth spurt)

Fixed:
Which cantilever has the LEAST success: I put a LATERAL abutment with a pontic replacing a central.
Base nobles are used for: long span FPDs
Why does an FPD keep failing? I put occlusal interferances
CD:
What causes angular cheilitis?
What is a sign of success for a CD? good peripheral seal
Pt management:
The ADA covers all except: Licensure
Modeling
Replacing amalgams with composites b/c of "allergy" this is covered under the which ADA code: Veracity
Which is a controlled stimulus in the dental office? fear, anxiety, DENTAL CHAIR (that's the only one that
made sense)
I had an incidence question: the answer was 100/1000 but I had to read the question a couple times b/c of
the way it was worded so BE CAREFUL.
I had some other pt mgmt questions but they weren't really hard...but there were terms that I wasn't familiar
with but just read carefully and make educated answer choices.
PICS:
Bell's Palsy
Pterygomaxillary fissue
Nutrient canal
Basal cell carcinoma
RL under the inf alveolar canal...but it wasn't called STAFNE's i put salivary duct something that's the only
one that made sense and I THINK (I haven't looked it up) but it maybe another name for stafne's bone
defect...??
Oral Path:
Lisch nodules, axillary pigmentation...Neurofibromatosis aka von reckinghausen
highest recurrent rate: OKC
highest risk of malignancy: i put osteomas...?
Ortho:
After ortho why does rotation occur again: apical fibers, oblique fibers, neural something, TRANSCEPTAL
FIBERS was my choice
Pharm:
What is the reversal for diazepam (benzodiapine): Flumazenil
If a pt wants pain therapy for 8hrs what do you give? naproxen
Pain med for 5yo? acetoaminophen
What should you avoid with ginseng? warfarin..?
INR is a test for what (I had this question twice, it was asked 2 diff ways)
Implants:
Distance between implants: 3mm
what temp will cause necrosis after 1-5min?: 57,43,47 deg Celcius
what is the LEAST likely cause of necrosis?: HIGH TORQUE
Ok so this is all I can remember off the top of my head. If I think of some more I'll repost.
~A!
Oh and LAST BUT NOT LEAST PRAY and BELIEVE!!!!!!

You might also like